SEARCH (Type the topic here)

Vascular Surgery MCQ

1. A 55-year-old woman gives a history of tiredness, aching, and a feeling of heaviness in the left lower leg for the past 3 months. These symptoms are relieved by leg elevation. She is also awakened frequently by calf and foot cramping, which is relieved by leg elevation, walking, or massage. On physical examination there are superficial varicosities, nonpitting edema, and a slightly painful, 2 cm. diameter superficial ulcer 5 cm. above and behind the left medial malleolus. What is the most appropriate diagnosis?
A. Isolated symptomatic varicose veins.
B. Superficial lymphatic obstruction.
C. Deep venous insufficiency.
D. Arterial insufficiency.
E. Incompetent perforating veins.
Answer: C


DISCUSSION: The most common symptoms associated with venous insufficiency are aching, swelling, and night cramps of the involved leg, which often occur after periods of sitting or inactive standing. Leg elevation frequently provides relief of symptoms due to venous insufficiency, while it increases pain due to arterial insufficiency. Although edema can occur with varicose veins alone, usually it is associated with deep venous abnormalities and incompetent perforators. In such cases, ulcers usually are located above and posterior to the malleoli, reinforcing their relationship with perforator abnormalities. The ulcers associated with arterial insufficiency may occur anywhere on the lower leg, eventually penetrate the fascia, and are more painful than venous ulcers.





2. The best treatment plan for the patient described in the preceding question should include:
A. Varicose vein ligation and stripping as soon as possible.
B. Ulcer débridement, vein stripping, and skin grafting.
C. Ligation of the medial perforating veins.
D. Transposition of saphenous vein valve.
E. Leg elevation, external Unna boot support, and ambulation without standing.
Answer: E


DISCUSSION: Operative treatment of venous insufficiency is in most instances an adjunct after failure of aggressive conservative management. Leg elevation, active exercise, and elastic compression form the cornerstones of nonoperative management. The goals of compression are to relieve symptoms and reduce swelling. The indications for superficial vein ligation and stripping are moderate to severe symptoms without other signs of deep venous insufficiency. If ulceration persists despite appropriate conservative management, ligation of the underlying incompetent perforators helps ulcer recurrence, and split-thickness skin grafting provides immediate coverage and healing of the ulcer. The patient must, however, comply with a program of external stocking support and prevention of leg edema since the underlying venous pathophysiology remains and ulcers tend to recur.




3. In patients who develop a documented episode of deep venous thrombosis (DVT) the most significant long-term sequela is:
A. Claudication.
B. Recurrent foot infections.
C. Development of stasis ulcer.
D. Pulmonary embolization.
E. Diminished arterial perfusion.
Answer: C


DISCUSSION: The increased hydrostatic pressure from incompetent venous valves following DVT predisposes to erythrocyte extravasation, hemosiderin deposition, and brown pigmentation around the ankle. Although the edema that occurs with deep venous insufficiency can predispose to skin infections, these usually are located about the ankle and resolve with adequate short-term care. When patients with a history of DVT are followed beyond 10 years, as many as 80% ultimately develop venous stasis ulcers. While there may be recurrence of DVT in a minority of patients, the incidence of pulmonary embolization is no greater than with the initial episode. Even in a leg severely affected by venous stasis changes, the arterial circulation is unimpaired unless there is concomitant arterial obstructive disease.




4. A 28-year-old woman developed a painful thrombosis of a superficial varix in the left upper calf 2 days previously. After spending the 2 days in bed with her leg elevated, she felt better and the tenderness resolved; however, when out of bed she developed a twinge of right-sided chest pain when walking and a feeling of heaviness in the calf. Which treatment is most appropriate?
A. Check for leg swelling, tenderness, and Homan's sign, and obtain a Doppler ultrasound study.
B. Begin antibiotics for a probable secondary bacterial infection.
C. Order emergency venography, and if it is abnormal, begin heparin administration.
D. Begin ambulation and discontinue bed rest that probably caused muscle pain by hyperextension of the knee.
E. If there is no pain on dorsiflexion of the left foot reassure her, since a negative Homan's sign precludes the diagnosis of DVT.
Answer: C


DISCUSSION: Associated DVT may occur during treatment of superficial venous thrombosis, especially if the process is near the groin or popliteal fossa. Although a positive Homan's sign or calf, popliteal, or groin pain is suggestive of DVT, clinical examination alone may be incorrect in more than 50% of cases. Noninvasive tests, including Doppler ultrasonography, are accurate for diagnosing DVT in the thigh but are less dependable in the calf. Emergency venography performed on an outpatient basis remains the most accurate and cost-effective technique for diagnosing DVT of the calf veins. Because 85% of pulmonary emboli arise from the lower extremity, early diagnosis and aggressive treatment are important.




5. In a 55-year-old grocery store cashier with an 8-month history of leg edema increasing over the course of a work day, associated with moderate to severe lower leg bursting pain, the most appropriate investigative study or studies are:
A. Doppler duplex ultrasound.
B. Brodie-Trendelenburg test.
C. Ascending and descending phlebography.
D. Measurement of ambulatory and resting foot venous pressure.
E. Venous reflux plethysmography.
Answer: A


DISCUSSION: While the Brodie-Trendelenburg test was an early attempt to clinically evaluate valve competence and function, it is neither quantitative nor precise. The development of phlebography allowed anatomic delineation of normal and abnormal veins and, when used in combination with invasive measurement of venous pressures in the foot at rest and on ambulation, helped correlate the venous hypertensive state with postphlebitic changes. Noninvasive plethysmography to quantitate the degree of venous valvular incompetence was more easily accepted; however the combination of B-mode duplex ultrasound (to accurately locate the vein of interest) plus pulsed Doppler flow signal is now the “gold standard” for venous assessment.




6. Which of the following statements are true of pulmonary embolism?
A. Most cases occur postoperatively.
B. In the majority of patients pulmonary emboli are ultimately lysed in situ without the administration of pharmacologic agents.
C. The preferred therapy for most patients is intravenous heparin.
D. It is generally safe to give thrombolytic agents as early as 48 hours postoperatively.
Answer: BC


DISCUSSION: Although many patients develop pulmonary embolism postoperatively, the majority of such lesions reported in most series do not follow operation. These patients develop thromboembolism as a complication of an underlying condition such as congestive heart failure, cerebrovascular accident, malignancy, chronic infection, and a variety of other debilitating diseases. Generally, postoperative patients comprise approximately one third of those with pulmonary embolism. Serial pulmonary scans following pulmonary embolism generally show gradual clearing of the emboli with re-establishment of perfusion in the occluded vessels. Depending on the magnitude of the embolism, most patients show the clearing at the end of a month to 6 weeks. The presence of persistent congestive heart failure, chronic infection, and atelectasis retard thrombolysis. This dissolution of emboli is generally agreed to be caused by naturally circulating thrombolysins. In fewer than 1% of cases the emboli persist and often increase with the passage of time, with the development of chronic pulmonary embolism leading to severe respiratory insufficiency, chronic cor pulmonale, pulmonary hypertension, right ventricular failure, and death. The majority of patients with pulmonary embolism are managed by continuous intravenous heparin. Thrombolytic agents are generally reserved for the management of extensive thromboembolism in patients with a stable cardiovascular system. Thrombolytic agents are generally withheld from postoperative patients until at least the fifth postoperative day, or preferably later. Earlier administration of these agents is apt to produce bleeding at the operative site. While it may occasionally be indicated to proceed earlier, it is generally best to wait until the thrombi in the vessels divided at the time of the surgical procedure have become organized.




7. Which of the following can cause a radioactive pulmonary perfusion scan to demonstrate an appearance similar to that of acute pulmonary embolism?
A. Atelectasis.
B. Pneumonitis.
C. Pleural fluid.
D. Emphysematous bullae.
Answer: ABCD


DISCUSSION: It has been shown by routine radioactive pulmonary perfusion scans that atelectasis, pneumonia, pleural fluid, emphysematous bullae, and pulmonary embolism may reduce pulmonary arterial blood flow to the involved segment. For this reason, it is imperative simultaneously to obtain chest film to exclude any significant radiopacity, which is usually associated with any defect that causes diminished pulmonary vascular perfusion.




8. In an otherwise healthy male with previously normal pulmonary and cardiac function, how much of the pulmonary vascular bed must usually be occluded to produce an unstable cardiovascular state (shock)?
A. 10%.
B. 20%.
C. 40%.
D. More than 50%.
Answer: D


DISCUSSION: If the patient had normal cardiovascular and respiratory function before the onset of pulmonary embolism, experimental and clinical studies have documented the fact that more than 50% of the pulmonary circulation must be occluded to produce cardiovascular collapse or shock. This can be considered similar to ligation of the pulmonary artery during the course of pneumonectomy, as occlusion by this procedure is also tolerated well, without development of hemodynamic instability. While bronchoconstriction may reduce pulmonary function in the normally perfused lung after embolism, this effect is generally short lived, as demonstrated by pulmonary ventilation scans.




9. Lytic therapy in pulmonary embolism:
A. Should precede anticoagulation.
B. Can be considered for all patients.
C. Can be considered for hemodynamically unstable patients.
D. Is indicated for the majority of patients with documented pulmonary embolism.
Answer: C


DISCUSSION: Thrombolytic therapy in pulmonary embolism involves use of streptokinase or urokinase. This therapy is indicated for hemodynamic compromise from documented pulmonary embolism that has not responded to anticoagulation and inotropic support. Additionally, lytic therapy is contraindicated after neurosurgery or cranial trauma and in persons who have a history of internal bleeding or hemorrhagic cerebral infarction. The majority of patients with documented pulmonary embolism do not require lytic therapy.




10. The single most important indication for emergency pulmonary embolectomy is:
A. The likelihood of another episode of embolism.
B. The inability to determine whether the problem is acute pulmonary embolism or acute myocardial infarction.
C. The presence of persistent and intractable hypotension.
D. Pulmonary emphysema.
Answer: C


DISCUSSION: The likelihood of another episode of pulmonary embolism is not an indication for pulmonary embolectomy since recurrent embolism may also be managed nonoperatively in most cases. Myocardial infarction should definitely be ruled out before pulmonary embolectomy is considered. In the presence of intractable cardiopulmonary collapse the only measure apt to correct the condition is emergency pulmonary embolectomy employing extracorporeal circulation. In such instances emboli generally are massive, with more than 50% of the pulmonary arterial circuit being occluded. The presence of cardiac and/or respiratory insufficiency before the attack of embolism is important, as that is apt to mean that a smaller pulmonary embolism is present that can cause serious cardiovascular changes.




11. In prevention of the fat emboli syndrome the primary therapy can be accomplished by which of the following?
A. Systemic anticoagulation achieving a partial thromboplastin time greater than 50 seconds.
B. Intravenous administration of alcohol.
C. Prophylactic administration of methyl prednisolone.
D. Maintaining a serum albumin value greater than 3 gm. per 100 ml. in the days immediately following injury.
Answer: D


DISCUSSION: The mechanism producing fat emboli syndrome is primarily release of free fatty acids that produce a change in the capillary alveolar membrane. Albumin is the primary binder of fatty acids. With serum levels greater than 3 gm. per 100 ml., 99% of the circulating free fatty acid remains in the bound configuration. Alcohol, heparinization, and steroids have been shown to have no direct effect on fat emboli syndrome, and serious side effects are related to their use.




12. Significant tachypnea and hypoxia follow development of fat emboli syndrome, and the goal of ventilatory support should be:
A. Keeping the respiratory rate below 30.
B. Preventing respiratory alkalosis.
C. Reversing pulmonary shunting using positive end-expiratory pressure.
D. Maintaining an adequate total volume.
Answer: C


DISCUSSION: The primary defect is increased pulmonary shunting secondary to reduced functional residual capacity. Use of positive end-expiratory pressure reverses this and is the primary goal of therapy.




13. Which of the following statements about the role of the endothelium is/are correct?
A. Endothelial cells only mediate vasorelaxation.
B. Endothelial cell–derived nitric oxide (NO) is produced by a constitutive and an inducible NO synthase.
C. The anticoagulant properties of the endothelium reside in its barrier function.
D. A local renin-angiotensin system is found in the walls of arteries and veins.
E. There are no significant differences in the vasomotor characteristic of endothelial cells in the macro- and microcirculation.
Answer: BD


DISCUSSION: Endothelial cells have the ability to mediate both contractile (prostanoids, oxygen free radicals, endothelins, angiotensins, and uncharacterized endothelium-derived constriction factors) and relaxant (prostanoids, nitric oxide and nitric oxide–containing compounds, hyperpolarization factor) responses. NO is synthesized from the conversion of L-arginine to citrulline by at least two categories of enzymes— constitutive nitric oxide synthases (cNOS, predominantly membrane bound) and inducible nitric oxide synthases (iNOS, predominantly cytosolic), both of which are calcium- and calmodulin dependent. Constitutive NOS is present in endothelial cells, and following cytokine stimulation endothelial cells also synthesize iNOS. In addition to its basic barrier function, the endothelium regulates intravascular coagulation by three other separate, but related, mechanisms: participation in procoagulant pathways, inhibition of procoagulant proteins, regulation of fibrinolysis and production of thromboregulating compounds. There is a local renin-angiotensin system in the vessel wall of both vein and arteries, and this system is considered important in the maintenance of blood pressure, the control of hypertension, and the response of the vessel wall to injury. In contrast to the macrocirculation, the microcirculation has distinctive vasomotor characteristics. Vessels of the microcirculation are the conduits that are responsible for the local delivery and transfer of cell substrates and metabolites. The endothelium regulates the microvasculature, reacting to the metabolic needs of tissue; it is essential in organ autoregulation and in the responses of these microvasculatures to changes in local blood flow. Reactive hyperemia appears to be dependent on the immediate production of endothelium-derived cyclo-oxygenase products, predominantly prostaglandin E 2 (PGE 2), as opposed to PGI 2, in the short term.




14. Which of the following statements on smooth muscle cells is/are correct?
A. Smooth muscle cells can undergo pheotypic changes in response to injury.
B. Platelet-derived growth factor (PDGF) requires a progression factor to initiate smooth muscle cell growth.
C. NO can be produced by smooth muscle cells.
D. Changes in the composition of the extracellular matrix modulate smooth muscle cell growth.
E. Smooth muscle cells are the principal cell involved in the development of intimal hyperplasia.
Answer: ABCDE


DISCUSSION: Smooth muscle cells are the principal cells found in the media of a vessel. They are embedded in a matrix of connective tissue elements and provide mechanical and structural support to the vessel. In addition to their vasoreactive characteristics, smooth muscle cells are capable of synthesizing and secreting elements of the extracellular matrix, particularly proteoglycans. The exact mechanisms whereby smooth muscle cell proliferation is initiated, controlled, reduced, and eventually suppressed are not fully understood. Quiescent vascular smooth muscle cells are well-differentiated cells characterized by an abundance of contractile proteins, predominantly smooth muscle cell actin and myosin but little rough endoplasmic reticulum. Once activated, smooth muscle cells lose their differentiated state, acquire abundant endoplasmic reticulum, and commence the synthesis of extracellular matrix. After cytokine stimulation, smooth muscle cells synthesize iNOS and produce nanomoles of NO for at least 24 hours. Smooth muscle cell proliferation depends on the presence of PDGF, basic fibroblast growth factor (bFGF), and insulin-like growth factor 1 (IGF-1). IGF-1 acts as a progression factor for PDGF in smooth muscle cells, and both in vivo and in vitro there is synergism between these two growth promoters. Smooth muscle cell growth in the wall can be modulated by various extracellular matrix substances such as collagen (type V), several glycoproteins, and the glycosoaminoglycans. Both NO and prostacyclin inhibit cell proliferation. Intimal hyperplasia is a structural lesion that develops in injured blood vessels after injury and is the result of smooth muscle cell migration into and proliferation within the intima of a blood vessel.




15. Which of the following statements correctly characterizes the healing of prosthetic arterial grafts in humans?
A. Complete healing occurs within 3 months of graft implantation.
B. Complete healing occurs within 1 year of graft implantation.
C. Prosthetic grafts do not heal completely in humans.
D. Polytetrafluoroethylene (PTFE) grafts heal completely whereas Dacron grafts do not.
E. Dacron grafts heal completely but PTFE grafts do not.
Answer: C


DISCUSSION: Prosthetic grafts of any kind do not heal completely in humans as they do in experimental animals. Some healing occurs in the perianastomotic regions, but the majority of the graft's luminal surface never develops a living neointima; rather, it remains covered with a compacted layer of fibrin.




16. Which of the following adversely influence the patency of lower extremity autogenous vein grafts?
A. Poor arterial outflow from the distal anastomosis of the graft.
B. Diabetes.
C. Small-caliber (less than 4 mm. in diameter) veins.
D. Use of reverse, rather than in situ, grafting technique.
E. Grafts performed for limb salvage indications rather than claudication.
Answer: ACE


DISCUSSION: Small-caliber veins perform poorly as arterial substitutes. The long-term patency of infrainguinal vein grafts is better when the indication for operation is claudication rather than limb salvage and is improved when there is a good outflow tract. Reverse vein and in situ bypass have equal patency rates at all levels. Surprisingly, the presence of diabetes has not been shown to negatively affect the patency of autogenous vein infrainguinal arterial bypass.




17. Arterial autografts are:
A. Limited by the length of available artery.
B. When available, always appear to function superiorly to venous autografts.
C. Are the graft of choice for pediatric renal artery grafting.
D. Are performed infrequently.
E. Are immune from the fibrointimal hyperplasia that frequently complicates venous autografts and prosthetic grafts.
Answer: AC


DISCUSSION: Arterial autografts have limited but well-established clinical uses. They are the graft of choice for pediatric renal artery revascularizations, as they do not appear to be subject to the aneurysmal dilatation that complicates pediatric renal artery vein bypass. The internal mammary artery autograft provides superior patency for coronary artery bypass as compared with the saphenous vein and is thus a very common arterial autograft procedure. Radial artery autografts for coronary bypass are, however, rapidly narrowed by fibrointimal hyperplasia.




18. PTFE grafts:
A. Are a variant of a woven textile graft.
B. Provide patency superior to that with Dacron grafts for suprainguinal revascularization procedures.
C. Provide patency equal to that of autogenous saphenous vein for above-knee femoropopliteal bypass.
D. May be more resistant than Dacron grafts to pseudoaneurysm formation.
E. Are currently the prosthetic graft of choice for hemodialysis access.
Answer: DE


DISCUSSION: PTFE grafts are manufactured from a unique polymer extrusion process and are not textile grafts. For most surgeons they are currently the prosthetic graft of choice for hemodialysis access. For suprainguinal bypass, PTFE and Dacron provide comparable patency, although PTFE grafts may be more resistant than Dacron grafts to pseudoaneurysm formation. PTFE at all levels in comparable patients is inferior to good-quality saphenous vein for infrainguinal bypass.




19. Which of the following statements about the evaluation of arterial substitutes are correct?
A. A graft is considered to have continued primary patency, even if it requires revision, as long as it has not actually thrombosed.
B. Secondary patency refers only to grafts whose patency has been restored following an episode of thrombosis.
C. Patency figures should be derived from life table calculations.
D. Primary patency is the best indicator of the natural history of an arterial substitute.
E. Secondary patency is the best indicator of the natural history of an arterial substitute.
Answer: CD


DISCUSSION: Patency is the most important variable in the evaluation of the clinical effectiveness of an arterial substitute, and it should always be derived from life table calculations. Both primary and secondary graft patency are important. Primary patency is the best indicator of the natural history of a graft. A graft remains primarily patent as long as it has not thrombosed or had any graft- or anastomosis-directed procedures. Grafts are considered to be secondarily patent if patency has been restored after an episode of thrombosis or if, in order to maintain patency, operation on a patent graft is directed toward the graft itself or its anastomoses.




20. Which of the following statements about aneurysms of the sinus of Valsalva is/are true?
A. Aneurysms of the sinus of Valsalva are dilatations of the aortic sinuses that eventually rupture into a cardiac chamber, the pulmonary artery, or the pericardium.
B. The most common cause of an acquired sinus of Valsalva aneurysm is bacterial endocarditis.
C. Congenital aneurysms of the sinus of Valsalva usually cause symptoms long before they rupture.
D. The most common defect associated with congenital sinus of Valsalva aneurysms is aortic insufficiency.
E. The most common symptoms of sinus of Valsalva aneurysm rupture include symptoms caused by obstruction of the ventricular outflow tract, heart block, and embolization.
Answer: AB


DISCUSSION: A sinus of Valsalva aneurysm is a dilatation of the aortic sinus that eventually ruptures into a cardiac chamber, the pulmonary artery, or the pericardium. The cause may be either congenital or acquired, and the most common cause of an acquired sinus of Valsalva aneurysm is bacterial endocarditis. The vast majority of sinus of Valsalva aneurysms are asymptomatic before they rupture, but if symptomatic they present as obstruction to either the left or right outflow tract, heart block, or embolization. When sinus of Valsalva aneurysms rupture, they present with symptoms of an acute left-to-right shunt and these include dyspnea, palpitations, and chest pain. Finally, associated defects occur commonly with congenital sinus of Valsalva aneurysms, the most common being ventricular septal defect followed by aortic insufficiency.




21. Which statements about treatment for sinus of Valsalva aneurysms are correct?
A. Close observation is appropriate for patients who have an asymptomatic sinus of Valsalva aneurysm without rupture.
B. Patients with sinus of Valsalva aneurysms that rupture should undergo operative repair because progressive heart failure may well lead to death.
C. All patients with suspected sinus of Valsalva aneurysm ruptures need to undergo cardiac catheterization prior to operation.
D. The best operative approach for closure of a ruptured sinus of Valsalva aneurysm is a dual approach through the aorta and the chamber of entry of the fistula.
E. When a sinus of Valsalva aneurysm ruptures into the pericardium, emergency operation is required.
Answer: ABDE


DISCUSSION: When a found intact sinus of Valsalva aneurysm is asymptomatic, the patient should be followed closely for symptoms but does not usually require operative intervention without symptoms or rupture. A ruptured sinus of Valsalva aneurysm should be repaired by surgical intervention because the significant left-to-right shunt that occurs often leads to progressive heart failure and death. While it is true that cardiac catheterization was the gold standard, the accuracy of noninvasive transesophageal color-flow Doppler echocardiography has replaced it in some cases recently. Operative intervention is not emergent unless the aneurysm ruptures into the pericardial space. If this occurs, symptoms of cardiac tamponade are present and emergent operation is required. Finally, the best operative approach is a dual approach via the aorta and the chamber of entry of the fistula. The major advantage of this approach is that the fistula can be identified through the aorta into the chamber involved and closed securely from both sides. The aortic valve can be protected or replaced as necessary, and the ventricular septum can be inspected for a ventricular septal defect.




22. Transection of the thoracic aorta following trauma usually:
A. Is located just distal to the left subclavian artery.
B. Produces a false aneurysm.
C. Is fatal in 80% of cases.
Answer: ABC


DISCUSSION: Transection of the thoracic aorta may occur at multiple sites from the isthmus down into the abdominal aorta. Transection of the ascending aorta occurs but is uncommon. Those who survive the initial injury and do not rupture into the pleural space do so because the false aneurysm is supported by the adventitia and pleura. The majority of transections occur just distal to the left subclavian and the immediate mortality is about 80%.




23. Which of the following confirms the diagnosis of transection of the descending thoracic aorta?
A. Widened mediastinum.
B. Fractured first rib.
C. Left pleural effusion.
D. Positive aortogram.
E. All of the above.
Answer: D


DISCUSSION: The diagnosis of transected aorta should be suspected with severe injury especially if the patient is thrown from the vehicle. A widened mediastinum, fractured first rib, or a pleural effusion can each be seen with chest trauma that does not involve the thoracic aorta. The diagnosis is best made by the use of the aortogram even though the diagnosis can be made with CT, MRI, and occasionally echocardiography.




24. The following is/are true of a descending dissecting aortic aneurysm:
A. It originates distal to the subclavian artery.
B. It is usually found in hypertensive patients.
C. It may extend the entire length of the aorta.
Answer: ABC


DISCUSSION: Descending dissections are almost always seen in hypertensive patients. The diagnosis is made by locating the tear in the aorta distal to the left subclavian artery. Type III dissections may involve only the descending thoracic aorta but can extend the entire length of the aorta.




25. The optimal management of Type A or ascending aortic dissection includes:
A. Aortography.
B. Hemodynamic monitoring and frequent recording of blood pressure, urinary output, and neurologic status.
C. Emergency operation.
Answer: ABC


DISCUSSION: Type A dissections are Type I or Type II dissections that originate in the ascending aorta. These may be associated with aortic insufficiency or hypertension or hypotension depending upon whether or not there has been bleeding into the pericardial space and an element of tamponade and/or neurologic deficit has occurred. Aortography is the best method to establish this diagnosis. Patients with dissecting aortic aneurysms originating in the ascending aorta are considered as emergencies. The operation involves grafting of the ascending aorta to prevent rupture in the mediastinum or into the pericardium with correction of the aortic insufficiency while redirecting the blood into the true lumen.




26. Aneurysms of the ascending aorta may be caused by:
A. Type II aortic dissection.
B. Atherosclerosis.
C. Cystic medial necrosis.
Answer: ABC


DISCUSSION: Aneurysms of the ascending aorta are classically associated with Marfan's syndrome, cystic medial necrosis, and, in past years, syphilis. Today, aneurysm formation in the ascending aorta is secondary to the manifestations of degenerative disease occurring within the aorta and the aorta wall.




27. When complications occur after operating on a descending thoracic aorta, perhaps the most devastating is:
A. Recurrent nerve injury.
B. Bleeding with hemothorax.
C. Paraplegia.
D. Renal insufficiency.
Answer: C


DISCUSSION: Although the recurrent nerve can be stretched and hoarseness seen after operations in the area of the isthmus of the thoracic aorta and bleeding associated with aneurysm repair as well as renal insufficiency from the cross-clamping of the vessel above the renals, they are usually insignificant compared to the problem that occurs when paraplegia follows operation on the descending thoracic or thoracoabdominal aorta. Not only is there a loss of function in the legs, but bowel and bladder function are usually seriously affected. The psychological effects of paraplegia may also be devastating.




28. The most common risk associated with carotid artery aneurysm is:
A. Thrombosis of the aneurysm.
B. Embolization of mural thrombus.
C. Rupture of the aneurysm.
D. Compression of the hypopharynx.
Answer: B


DISCUSSION: Thrombosis of the aneurysm and internal carotid artery may occur through deposition of laminated clot within the aneurysm and kinking of the outflow tract of the internal carotid artery. However, this is a relatively uncommon clinical presentation for such an aneurysm. The majority of patients with carotid artery aneurysm present with transient ischemic attacks in the ipsilateral cerebral hemisphere secondary to embolization of laminated clot lining the aneurysm wall. If the aneurysm is left uncorrected, most patients ultimately suffer a stroke. Although rare, rupture of a carotid aneurysm is almost always fatal. Rupture usually occurs into the oral cavity or nasopharynx, and death occurs from suffocation. Aneurysms may present with symptoms of dysphagia secondary to pressure on neighboring cranial nerves and the lateral wall of the pharynx. Nevertheless, these presentations do not represent serious risks to the patient.




29. Risks associated with carotid artery aneurysms are treated most successfully by which of the following?
A. Proximal ligation.
B. Observation.
C. Resection and graft replacement.
D. Resection and reanastomosis.
Answer: CD


DISCUSSION: Proximal ligation was the first method of surgical treatment and was employed during the nineteenth and early twentieth centuries, but a high incidence of stroke was associated with its use. Gradual occlusion over several days with a Crutchfield clamp can be used in the rare instance when resection and restoration of flow cannot be performed. Reports from the first part of this century indicate that observation of symptomatic carotid aneurysms yielded a high incidence of stroke and subsequent death. The preferred management of carotid aneurysms is resection and either reanastomosis of the internal and common carotid arteries or interposition graft replacement. Large aneurysms frequently require interposition grafting. Most aneurysms can be removed successfully, and flow can be re-established by reanastomosis. Currently, this is being achieved with a combined operative stroke and mortality rate of less than 2%.




30. Which of the following statements about carotid body tumors are true?
A. Cells from which carotid body tumors arise normally sense changes in systemic blood pressure.
B. Most carotid body tumors are malignant and usually metastasize to the ipsilateral cerebral hemisphere.
C. Carotid body tumors are extremely vascular.
D. Carotid body tumors most frequently present as a palpable, painless mass at the carotid bifurcation.
Answer: CD


DISCUSSION: Cells of the carotid body function as receptors for changes in PO2, PCO2, and pH. Interestingly, carotid body tumors are more prevalent in persons living at high altitudes. Although carotid body tumor cells may have characteristics of malignancy, only about 5% of them metastasize. A carotid body tumor characteristically appears as a hypervascular oval mass situated at the carotid bifurcation and widening the angle between the origins of the internal and external carotid arteries. Although large or malignant carotid body tumors may cause dysfunction of the vagus and hypoglossal nerves and cause dysphagia from their mass effect on the hypopharynx, most tumors are recognized when they still have no associated symptoms and are only a painless mass over the area of the carotid bifurcation.




31. Treatment of carotid body tumors most frequently consists of:
A. Radical neck dissection, including the extracranial carotid artery.
B. Radiation therapy.
C. Resection of the common, internal, and external carotid arteries with interposition grafting.
D. Subadventitial dissection of the carotid bifurcation and simple excision of the tumor.
Answer: CD


DISCUSSION: Malignant carotid body tumors are uncommon. They are treated either by limited resection of the structures involved with the tumor or by radiation therapy. Although radiation therapy has been demonstrated to shrink some tumors, it should be used only for the rare tumor that is unresectable. Most tumors are benign and can be separated from the wall of the carotid artery without resection of the bifurcation. Such resection is possible because the tumor is located in a plane outside the media. Therefore, upon creation of a subadventitial plane, one can separate the tumor from the carotid artery bifurcation without entering the vessel.




32. The cause of subclavian arterial aneurysms is most often:
A. Sepsis.
B. A congenital defect.
C. Atherosclerosis.
D. Fibromuscular dysplasia.
Answer: C


DISCUSSION: The most common cause of subclavian aneurysms is atherosclerosis followed by trauma. The thoracic outlet syndrome may cause dilatation of the distal subclavian artery secondary to compression with poststenotic dilatation. Mural thrombi are quite common in these aneurysms and may cause embolism in an arm, a serious complication.




33. Of the visceral aneurysms, which is the most common?
A. Celiac.
B. Superior mesenteric.
C. Hepatic.
D. Splenic.
Answer: D


DISCUSSION: Splenic aneurysms are the most common visceral type. They are usually caused by medial degeneration of the arterial wall, although fibromuscular dysplasia can be a cause.




34. Aneurysms of the renal artery are most common:
A. At its origin from the aorta.
B. In the main renal artery or the bifurcation into the primary branches.
C. Within the kidney.
Answer: B


DISCUSSION: The majority of renal aneurysms are located either in the main renal artery or at the point of bifurcation into the branches. The appropriate treatment is resection of the aneurysm with restoration of continuity of the artery whenever possible, often with the use of a graft.




35. An aortic abdominal aneurysm was first successfully resected by:
A. Matas.
B. Linton.
C. Dubost.
D. None of the above.
Answer: C


DISCUSSION: Charles Dubost was the first to successfully correct an aortic abdominal aneurysm by resecting the entire aneurysm and replacing it with an aortic homograft. In his paper he presented preoperative as well as postoperative arteriograms demonstrating correction of both the aortic aneurysm and severe stenosis of the left iliac artery.




36. Evaluation of the natural history of abdominal aortic aneurysms in patients who are followed without any surgical procedure indicates that:
A. Approximately 20% are alive at the end of 5 years.
B. Seventy-five per cent of patients succumb by the end of the first year.
C. Aortic rupture is quite common in this group, occurring in more than half by the second year.
D. None of the above.
Answer: A


DISCUSSION: In the classic study of Estes in 1951, during an era before surgical correction, the survival at 5 years of patients with aortic abdominal aneurysms was 20%.




37. The appropriate treatment in most situations of an aortic abdominal graft that has become infected is:
A. Intravenous antibiotics and observation for future complications.
B. Catheter drainage at the site of infection.
C. Replacement of the infected graft with another prosthetic graft.
D. Excision of the entire graft and insertion of axillobifemoral grafts.
Answer: D


DISCUSSION: Many forms of management have been tried and occasionally recommended. The most appropriate treatment for most patients with an infected graft is removal of the entire graft, closure of the iliac or femoral arteries distal to the insertion of the graft, and placement of axillobifemoral grafts.




38. In a patient with an abdominal aortic aneurysm and a history of several previous abdominal procedures for release of dense peritoneal adhesions causing episodes of intestinal obstruction, consideration should be given to which one of the following at operation?
A. Cardiopulmonary bypass.
B. An incision from the xiphoid process to the symphysis pubis.
C. Incision in the left flank with a retroperitoneal approach.
D. An axillobifemoral graft.
Answer: C


DISCUSSION: A retroperitoneal approach has been recommended, particularly for patients who have had previous abdominal procedures when peritoneal adhesions might represent a serious technical problem. In these circumstances, the retroperitoneal approach can be quite satisfactory, especially in the management of small aneurysms and those located in the midportion of the abdominal aorta between the renal arteries and aortic bifurcation. However, prolonged postoperative incision pain may be a complication in some patients with this incision.




39. After emergency correction of an aortic abdominal aneurysm, the two most common causes of mortality are:
A. Acute renal insufficiency.
B. Severe hemorrhage from dehiscence of the suture line postoperatively.
C. Myocardial infarction.
D. Infection of the graft.
Answer: AC


DISCUSSION: Although all these complications may follow correction of an aortic abdominal aneurysm, the most common ones are fatal complications from myocardial infarction and acute renal insufficiency.




40. The incidence of inflammatory aortic abdominal aneurysms with dense periaortic adhesions and possible involvement of adjacent structures such as the duodenum, renal vein, and ureter is approximately:
A. 2%.
B. 10%.
C. 25%.
Answer: B


DISCUSSION: Increased attention is being given to inflammatory aortic abdominal aneurysms. These are characterized by aneurysms surrounded by dense periaortic inflammation and at times involvement of surrounding structures such as the duodenum, renal vein, and ureter. This problem occurs in some 7% to 10% of patients undergoing aneurysmectomy.




41. Which of the following statements about true femoral artery aneurysms is/are correct?
A. All three layers of the blood vessel wall are involved in true aneurysms.
B. There is a very high association with aortoiliac and popliteal aneurysms.
C. Femoral artery aneurysms occur bilaterally in about 10% of cases.
D. Type I femoral artery aneurysms involve the orifice of the deep femoral artery.
E. The most common complication of femoral aneurysms is rupture.
Answer: AB


DISCUSSION: By definition, a true aneurysm involves the intima, media, and adventitia. Femoral aneurysms are very often associated with other aneurysms, ranging from 70% to 95% in most reports. They are most commonly associated with aortoiliac or popliteal aneurysms. Approximately 50% of femoral aneurysms are bilateral. Type I femoral aneurysms are limited to the common femoral artery; type II aneurysms involve the orifice of the deep femoral artery. Rupture of femoral artery aneurysms is rare. The most common complications are due to thrombosis, embolism, or local compression.




42. Which of the following statements about false aneurysms of the femoral artery is/are correct?
A. The incidence of iatrogenic false aneurysms has increased in recent years.
B. Arteriography is the most useful study for diagnosis of iatrogenic femoral aneurysms.
C. Ultrasound-guided compression of iatrogenic false aneurysms is usually successful in achieving thrombosis.
D. Femoral anastomotic aneurysms usually involve the proximal anastomosis of a prosthetic infrainguinal bypass.
E. Rupture is the most common complication associated with femoral anastomotic aneurysms.
Answer: AC


DISCUSSION: With the introduction of invasive percutaneous procedures such as angioplasty, valvuloplasty, atherectomy and coronary stenting, the incidence of iatrogenic false aneurysms has risen to about 0.6% to 1.0% in recent years. Color-flow duplex ultrasound is the best test for diagnosis of these aneurysms, owing to its ability accurately to delineate the anatomy. Additionally, compression of these false aneurysms using ultrasound guidance is very effective in achieving thrombosis. Femoral anastomotic aneurysms are most commonly seen after aortofemoral bypass grafting and only rarely are associated with infrainguinal procedures. Thromboembolic complications are the most common complication of these lesions.




43. Which of the following statements about popliteal artery aneurysms is/are correct?
A. They are the most common site of peripheral artery aneurysms.
B. For a patient with an abdominal aortic aneurysm the risk of a popliteal aneurysm is approximately 50%.
C. For a patient with a popliteal artery aneurysm the risk of a contralateral popliteal aneurysm is approximately 50%.
D. Popliteal artery aneurysms most commonly present with local symptoms secondary to compression of the adjacent vein or nerve.
E. Arteriography is the most accurate test for the diagnosis of popliteal artery aneurysm.
Answer: AC


DISCUSSION: Although rare, popliteal artery aneurysms are the most common site of peripheral artery aneurysms. While the incidence of associated aneurysms in patients with popliteal aneurysms is high, the opposite is not true. The likelihood of a patient with an abdominal aortic aneurysm having a popliteal artery aneurysm is less than 10%. About 50% of popliteal aneurysms are bilateral. The most common presenting symptom in patients with popliteal aneurysms is leg ischemia, secondary to either thrombosis or embolism. Local symptoms due to the aneurysm are relatively rare. Ultrasonography is the best test for diagnosis of a popliteal aneurysm. Arteriography should be reserved for patients undergoing operative repair.




44. Which of the following statements about management of popliteal artery aneurysms is/are correct?
A. All symptomatic aneurysms should be treated with surgery.
B. The most common operation is excision of the aneurysm with arterial reconstruction.
C. Thrombolytic therapy may be useful when there is thrombosis of the aneurysm and the distal runoff vessels.
D. The results for surgery for asymptomatic aneurysms are better than those for symptomatic ones.
E. The long-term results with prosthetic grafts are equivalent to those of autogenous vein grafts.
Answer: ACD


DISCUSSION: It is generally felt that all symptomatic popliteal aneurysms should be treated with surgery. Treatment of asymptomatic aneurysms is more controversial; some advocate repair of all of these as well. A reasonable approach is to consider surgery for asymptomatic aneurysms larger than 2 cm. Aneurysm ligation with bypass is the most commonly performed operation for popliteal aneurysm, excision being reserved for aneurysms that cause local compressive symptoms. Thrombolytic therapy, given either before or during operation, is useful in clearing the tibial and pedal vessels in cases of acute thrombosis. The patency and limb salvage rates are better for asymptomatic lesions than for symptomatic ones. Autogenous vein is superior to prosthetic material and is the conduit of choice for ligation and bypass of popliteal aneurysms.




45. Which of the following statements about thrombo-obliterative disease of the aorta and its branches are correct?
A. The most common cause of obstructive disease is thrombi.
B. Atherosclerosis is the most common pathologic cause of arterial obstruction.
C. Lesions occur with greater frequency at the origin of vessels from the aorta.
D. Obstructive lesions are preferentially managed by endarterectomy.
Answer: BC


DISCUSSION: Thrombo-obliterative disease of the aorta and its branches is primarily due to atherosclerosis. The lesions occur most frequently at the origin of blood vessels and are usually localized and short. Surgical management consists primarily of bypass grafts since endarterectomy is often followed by later reocclusion with reappearance of symptoms.




46. Which of the following statements about Takayasu's disease is/are correct?
A. Atherosclerosis is restricted to the ascending aorta and innominate artery.
B. It primarily affects patients of Asian descent.
C. It is a nonspecific arteritis affecting the thoracic and abdominal aorta and its major branches.
D. The disorder is also characterized by systemic symptoms, including fever, malaise, arthritis, and pericardial pain.
E. Surgical bypass of the involved vessel should be undertaken in nearly all patients, since the results are excellent.
Answer: BCD


DISCUSSION: While it may affect others, Takayasu's disease occurs primarily in those of Asian descent and usually attacks young females. It is basically a nonspecific arteritis involving all layers of the arterial wall with proliferation of connective tissue and degeneration of elastic fibers. Clinical manifestations include fever, malaise, arthritis, and pericardial pain. Some believe that it may be an autoimmune disease, and steroids have been effective in some patients. Late manifestations include ischemia of both cerebral and upper extremity circuits. While surgical management is occasionally successful, it usually is not recommended except for patients with disabling symptoms.




47. A 65-year-old man complains of having had slurred speech and no motor function or sensation of his right hand for 15 minutes. A left carotid bruit is heard in the neck. Which of the following diagnostic studies should be done?
A. Carotid duplex scan.
B. Electroencephalography (EEG).
C. Carotid arteriography.
D. Computed tomography (CT) of the brain.
Answer: CD


DISCUSSION: Although a carotid duplex scan provides valuable information on the presence of significant carotid artery disease at its bifurcation, it cannot be used as the final test of the circulation to the brain in this particular case. Carotid duplex scanning has its greatest value in the assessment of asymptomatic patients who have cervical bruits. In that circumstance, it can provide information on which the decision for further workup can be based. When symptoms suggest a transient ischemic attack, no noninvasive study provides a complete evaluation and arteriography is mandatory. EEG is a valuable test for evaluating patients when seizure activity is suspected. In this patient, an EEG would be valuable only if all other diagnostic studies had been unrevealing. An arteriogram is mandatory in this patient to adequately evaluate the entire extracranial and intracranial cerebral circulation. If a duplex scan had been performed and had revealed no disease, an arteriogram would still be necessary to exclude brachiocephalic trunk disease as a source of emboli. Likewise, intracranial narrowing would also be missed by such a noninvasive study. For these reasons, duplex scanning might be considered redundant and inadequate in these circumstances. CT of the head would be imperative in the evaluation of this patient. Its primary role is to exclude cerebral infarction, even in the presence of transient symptoms. It also distinguishes an ischemic cerebral event from an intracranial hemorrhage and rules out other potential causes of the symptoms, such as a brain tumor, other space-occupying intracranial lesions, and arteriovenous malformation.




48. Carotid artery occlusive disease most often produces transient ischemic attacks or stroke by which of the following mechanisms?
A. Reduction of flow to the affected area of the brain through stenotic or occluded vessels.
B. Embolization of atheromatous debris and/or clot with occlusion of intracranial branches of the carotid artery.
C. Thrombosis and propagation of the clot into the intracranial branches.
D. All of the above are equally common.
Answer: B


DISCUSSION: The collateral network to the brain is extensive. Collateral flow to an area supplied by a carotid artery is provided by the contralateral carotid artery and vertebrobasilar system around the circle of Willis, by the external carotid artery branches around the eye, and by direct intracerebral connections between the anterior cerebral arteries. For these reasons, low-flow cerebral symptoms are extremely rare, even when carotid lesions are present bilaterally. Approximately 70% of all cerebral symptoms produced by carotid artery occlusive disease are embolic in origin. The surface of an atherosclerotic lesion at the carotid bifurcation is thrombogenic and acts as a nidus for the accumulation of platelet-rich thrombi. Similarly, the interior of the plaque can degenerate and rupture into the lumen, embolizing its contents into the distal bed of the carotid circulation. The final event in the progression of an atherosclerotic carotid lesion is total thrombosis. Because there are no extracranial branches of the internal carotid artery, the thrombus propagates distally. If the propagation of the clot stops at the first major intracranial branch (the ophthalmic artery) and does not disturb collateral flow through it, no cerebral ischemic event may occur. When propagation continues into the intracranial branches, a massive stroke may occur. This mechanism accounts for a minority of strokes.




49. The majority of patients with “subclavian steal” syndrome have which of the following conditions?
A. Reversed flow in the involved vertebral artery.
B. Disabling neurologic symptoms.
C. Upper extremity claudication.
D. Decreased systolic blood pressure in the ipsilateral arm.
Answer: AD


DISCUSSION: Subclavian steal syndrome results from occlusion of a subclavian artery, rarely the innominate, with decreased systolic pressure distal to this obstruction. This causes blood to flow up the contralateral vertebral area and across the basilar artery (from which more blood is “stolen”) as it courses down (in a retrograde manner) the ipsilateral vertebral artery to help supply that subclavian artery. Most patients with this phenomenon are asymptomatic, although limb weakness and paresthesias or symptoms of vertebral basilar insufficiency may occur. Strokes do not occur in patients with subclavian disease alone. Most affected patients, however, have associated atherosclerotic disease of other extracranial arteries, particularly the carotid vessels, which may contribute to symptoms of cerebral ischemia.




50. Which of the following treatments is/are appropriate for symptomatic subclavian steal syndrome?
A. Subclavian endarterectomy.
B. Carotid-subclavian bypass.
C. Subclavian-carotid transposition.
D. Intra-arterial streptokinase.
Answer: BC


DISCUSSION: Because most patients with the abnormal flow phenomena demonstrated in subclavian steal syndrome are asymptomatic, surgical treatment is not necessary. Furthermore, some patients with subclavian steal have symptoms caused by other extracranial arterial disease and benefit from therapy directed at carotid disease. For patients whose symptoms are attributed to the subclavian steal phenomenon, a variety of surgical procedures have been proposed. Relief of symptoms has been obtained in 70% to 80% of these patients. Currently, both carotid-subclavian bypass and subclavian-carotid transposition are considered acceptable.




51. Which statements about thrombo-obliterative disease of the terminal abdominal aorta (Leriche syndrome) are true?
A. It is characterized by a combination of atherosclerotic and thrombotic occlusion of the terminal aorta.
B. It is characterized by acute thrombosis of the terminal aorta.
C. It requires emergency revascularization when the diagnosis is made.
D. It is often associated with distal obstructive lesions.
E. Patients often show signs of ischemia in the legs, and males may have difficulty maintaining a stable erection.
F. The preferred surgical procedure is bypass with a prosthetic graft.
Answer: ABEF


DISCUSSION: Leriche syndrome usually affects males 35 to 60 years of age. It is caused by a combination of thrombosis superimposed on atherosclerosis. The symptoms are usually those of ischemia of the legs, including weariness or claudication, atrophic changes, and the inability to maintain a stable erection owing to inadequate arterial flow to the penis. Distal sites of segmental occlusion are not uncommon. This disorder is often tolerated well for 5—and even 10—years, but ultimately serious symptoms of ischemia require operation. The procedure of choice is a bypass with a prosthetic graft from the aorta to the iliac or femoral arteries bilaterally.




52. Following surgical correction of Leriche syndrome, which of the following are true?
A. Gangrene is usually prevented, but symptoms of claudication persist.
B. The symptoms of claudication are usually improved.
C. Sexual function is improved in some patients.
D. Patients who continue to smoke following surgical correction have a higher incidence of reocclusion.
E. Prevention of damage to the sympathetic and parasympathetic nerves in the periaortic region reduces the postoperative incidence of retrograde ejaculation.
Answer: BCDE


DISCUSSION: Following corrective operation, patients with a history of claudication as well as ischemic manifestations are usually considerably improved. Those who have one or more patent internal iliac (hypogastric) arteries are more likely to have improved sexual function. It is important that patients cease smoking following bypass grafts since there is objective evidence that thrombosis of grafts is appreciably increased by continuance of smoking. Minimal dissection in the region of the abdominal aorta is important in minimizing damage to the sympathetic and parasympathetic nerves, since injury is related to the incidence of postoperative retrograde ejaculation.




53. In a patient who has chronic, complete occlusion of a common iliac artery, which of the following are true?
A. The primary symptom is claudication of the calf muscles.
B. Symptoms are usually claudication of the thigh and calf.
C. The decision as to whether or not to operate can be based on clinical examination findings.
D. Collateral iliac arterial vessels are prevalent.
E. Balloon angioplasty is appropriate in some patients.
Answer: BDE


DISCUSSION: Occlusion of the common iliac artery is usually associated with claudication of the thigh and calf. Arteriography should be done to establish the diagnosis and to assess the peripheral arterial system. Arteriography is also quite helpful in deciding whether or not balloon angioplasty is indicated, since it can be used successfully in some patients. Collateral vessels are usually apparent on the arteriogram. Bilateral involvement of the iliac vessels is quite common, and if symptoms are not present at the outset they may develop later. The preferred surgical management is bypass grafts from the aorta to the patent distal circulation.




54. Which of the following does not describe intermittent claudication?
A. Is elicited by reproducible amount of exercise.
B. Abates promptly with rest.
C. Is often worse at night.
D. May be an indication for bypass surgery.
Answer: C


DISCUSSION: Intermittent claudication characteristically is elicited by a relatively reproducible amount of exercise, and it is promptly relieved by cessation of that exercise. Furthermore, it is usually felt in a defined muscle group such as the calf muscles. Rest pain, indicating impending limb loss, is felt in the toes and forefoot as opposed to a muscle group, and this pain often begins at night. Claudication, when sufficiently disabling, may be an indication for revascularization, particularly if it interferes with gaining a livelihood.




55. In terms of long-term graft patency, the best results in the femoral tibial bypass position have been achieved with:
A. A modified human umbilical cord graft.
B. Polytetrafluoroethylene (PTFE [Gore-Tex]).
C. Saphenous vein allograft.
D. Segments of greater and lesser saphenous and cephalic veins spliced together.
Answer: D


DISCUSSION: Particularly for distal bypass grafting, the autogenous saphenous vein is the graft material of choice. When this is not available intact, several segments of this vein, as well as lesser saphenous and upper extremity veins, can be used for creating a graft of sufficient length. When the distal anastomosis is below the popliteal artery autogenous tissue grafts are clearly superior to synthetic ones.




56. Which of the following statements about femoral popliteal bypass grafting is/are true?
A. Long-term graft surveillance by duplex scanning has no effect on graft patency rates.
B. Graft failure and amputation occur in half the patients within 5 years.
C. If grafting is successful, long-term mortality is improved.
D. Patency rates of 80% to 90% at 1 year currently are expected.
Answer: D


DISCUSSION: Graft surveillance and repair of acquired defects in the vein before graft thrombosis improve long-term patency rates. Patients who require lower extremity bypass for limb salvage have poor 5-year survival because of concomitant coronary and cerebrovascular disease. Current graft patency rates approach 90% at 1 year.




57. Which of the following statements about percutaneous renal artery transluminal angioplasty (PRTA) are true?
A. Patients with renovascular hypertension are usually cured after successful PRTA.
B. Patients with renovascular hypertension due to atherosclerosis are more likely to benefit from PRTA than those in whom it is due to fibromuscular dysplasia.
C. PRTA of ostial atherosclerotic lesions is more successful than PRTA of nonostial lesions.
D. PRTA is associated with a higher morbidity and mortality than angioplasty for peripheral vascular disease.
Answer: D


DISCUSSION: The results of treatment for renovascular hypertension depend on the nature of the obstructing lesion and its anatomic location. Treatment of obstruction due to fibromuscular dysplasia is technically successful in 90% of patients and helps to control hypertension in almost 85%. However, fewer than 60% are cured. The results of treating atherosclerotic causes are substantially worse. Approximately 60% of patients show improvement in blood pressure control, but fewer than 25% are cured. The results of treating nonostial atherosclerotic lesions are better than those for ostial lesions. Unfortunately, the majority of patients with renovascular hypertension have ostial atherosclerotic lesions as the obstructing lesion. The complication rate associated with PRTA is substantially higher than that for angioplasty of vascular disease of the lower extremities, averaging almost 20% in patients with atherosclerotic lesions. Renal insufficiency is more likely. The mortality averages 1%, but higher rates are reported.




58. True statements about percutaneous transluminal angioplasty (PTA) of peripheral vascular lesions include which of the following?
A. PTA of iliac lesions is more often successful than PTA of femoral artery lesions.
B. PTA of iliac occlusions produces results similar to PTA of iliac artery stenoses.
C. PTA of infrapopliteal occlusive disease is associated with an increased rate of vasospasm, which can cause thrombosis.
D. A short, singular arterial stenosis is the optimal situation for a successful angioplasty.
Answer: ACD


DISCUSSION: Clinical results using PTA to treat occlusive vascular disease of the lower extremities depend on the morphologic nature of the obstruction as well as the anatomic location. PTA of iliac lesions is technically successful in approximately 90% of patients with 3-year patency rates of approximately 80%. PTA of superficial femoral artery lesions is technically successful in 75% to 85% of patients with 3-year patency rates of approximately 60% to 70%. PTA of infrapopliteal vessels is associated with increased risk of vasospasm, which can cause thrombosis. Improvements in the technique and pharmacologic treatment of vasospasm have reduced the risk of vessel thrombosis to less than 10%. Although some series have reported PTA of infrapopliteal lesions in patients with claudication, most authors recommend treating only patients with threatened limbs. Ideal patients for PTA are those with stenotic lesions less than 5 cm. long; the best results occur in the shortest lesions. Although total occlusions and multiple stenoses have been treated successfully, there are fewer technical successes, and long-term patency rates are substantially lower than in the treatment of stenotic lesions.




59. Advantages of PTA, as compared with surgical revascularization, include which of the following?
A. Decreased initial cost, shorter convalescence, and earlier return to full activity.
B. Because PTA is performed under local anesthesia, it is applicable to a greater number of patients with peripheral vascular disease.
C. PTA is more durable and requires fewer subsequent procedures.
D. Repeat PTA is well-tolerated, morbidity is equivalent to that for the initial procedure, and success rates are comparable to those expected with the initial procedure.
Answer: AD


DISCUSSION: The primary advantages of PTA over surgical revascularization are the lower initial cost of the procedure, decreased hospital stay, earlier return to full activity, lower morbidity, and the ability to repeat the procedure without markedly increasing patient morbidity or compromising clinical results. The procedure can be performed in patients who would have a high operative risk related to coexisting medical conditions; however, angioplasty is not applicable to the majority of patients with vascular occlusive disease because of the presence of long stenoses, occlusion, or multiple lesions in the vascular system. Restenosis and progression of disease in the vascular system also limit the long-term results of PTA and are the cause of a greater need for subsequent procedures.




60. Which of the following statements are true?
A. All arterial injuries are associated with absence of a palpable pulse.
B. Preoperative arteriography is required to diagnose an arterial injury.
C. The presence of Doppler signals indicates that an arterial injury has not occurred.
D. Patients with critical limb ischemia have paralysis and paresthesias.
E. In all patients with multiple trauma, arterial injuries should be repaired before other injuries are addressed.
Answer: D


DISCUSSION: Although arteriography may be required to diagnose partially severed or damaged arteries with persistent flow, it is not required for the diagnosis of most injuries. In a patient who is not in shock, a diminished pulse on physical examination indicates an arterial injury; however, patients with only partial severance or intimal injury may maintain axial flow through the injury and distal pulses will be palpable. In a patient who has adequate collateral flow, Doppler signals may be heard even in the presence of complete occlusion of an axial artery. These patients may have audible flow, but their Doppler-derived pressure will be decreased. Patients who do not have adequate collateral flow present with critical ischemia manifested by paralysis and paresthesias, the latter in a stocking or glove distribution. Patients who have neither paralysis nor paresthesia do not have immediately limb-threatening injuries and, for them, arterial reconstruction can be deferred until more acute life- and limb-threatening lesions have been treated.




61. A patient presents with a gunshot wound of the mid-neck. Although drunk, he exhibits no lateralizing neurologic signs. After control of his airway is achieved, he is taken directly to the operating room for control of hemorrhage. The common carotid artery has a 2-cm. destroyed segment. There is also a major esophageal injury. The best treatment for this carotid injury is:
A. Vein graft replacement of the common carotid artery.
B. Ligation of the common carotid artery.
C. Ligation of the common carotid artery proximally with a subclavian carotid bypass.
D. Ligation of the common carotid artery with sympathectomy.
E. Prosthetic graft replacement of the common carotid artery.
Answer: A


DISCUSSION: The treatment of carotid artery injury in a patient without neurologic deficit consists of reconstruction of the carotid. This patient had no neurologic deficit preoperatively and should have a good result. Autogenous material is preferred over prosthetic material. Ligation without reconstruction may be tolerated in the common carotid artery but is not recommended. Subclavian carotid bypass has no advantage over reconstruction unless it can be performed in a clean operative field. Cervical sympathectomy is not performed in these circumstances. Although the presence of an esophageal wound means that there is a potential for infection, it should not deter reconstruction with autogenous material. This potential for infection, however, should alert the physician to the possible development of false aneurysm in the postoperative period. If an infected false aneurysm develops, alternative methods of carotid reconstruction must be utilized.




62. Which of the following statements about iatrogenic arterial injuries are true?
A. Femoral artery pseudoaneurysms occurring after arteriography require urgent operative intervention.
B. Symptomatic axillary sheath hematomas require urgent operative intervention.
C. Arterial occlusions after catheterization occur more commonly in the femoral artery than in the brachial artery.
D. The Allen test identifies patients with an incomplete palmar arch.
Answer: BD


DISCUSSION: Small pseudoaneurysms after arterial puncture often close spontaneously. Duplex-guided compression may be used to cause thrombosis of the pseudoaneurysm. Surgery is usually reserved for large or expanding pseudoaneurysms that are not successfully occluded by duplex-guided compression. On the other hand, axillary sheath hematomas can cause permanent nerve injury and require urgent surgical decompression. Occlusions after catheterization are more common in smaller arteries such as the brachial artery, but ischemic signs and symptoms are more likely with larger vessel occlusions. The Allen test should be performed before all radial artery cannulations to show that there is an adequate palmar arch.




63. A 35-year-old man involved in a motor vehicle accident presents with a knee dislocation that is easily reduced. Radiography of the knee shows no fracture. Which of the following statements about his treatment are true?
A. If he has normal pulses he can be discharged.
B. If he has normal pulses he requires either close observation or arteriography.
C. If he has absent distal pulses and severe ischemia he should undergo arteriography in the radiology suite.
D. A popliteal vein injury is best treated with ligation.
E. A popliteal artery injury should be repaired with the ipsilateral saphenous vein if available.
Answer: B


DISCUSSION: Knee dislocations commonly cause popliteal vessel injury. Early after injury, flow may be sustained in the injured popliteal artery; therefore, these injuries require close observation or arteriography. When an occluded popliteal artery is evident clinically, immediate repair is indicated and arteriography is unnecessary. Repair should ideally utilize the contralateral saphenous vein, to maintain optimal venous return in the injured leg. Injured popliteal veins should be repaired if at all possible. Ligation may jeopardize the arterial repair and increase the likelihood of chronic venous insufficiency and the risk of pulmonary embolism.




64. A 24-year-old man is involved in an industrial accident in which he sustains a crushed pelvis. Diagnostic peritoneal lavage is positive. At exploration, a large pelvic hematoma is found. What is the best treatment?
A. Explore all the major arteries and veins of the pelvis and surgically control the bleeding if possible.
B. Do not explore the pelvic hematoma. Close the abdomen and apply a MAST suit.
C. Do not explore the pelvic hematoma. Apply a pelvic fixator and send the patient to radiology for possible embolization of bleeding pelvic vessels.
D. Use sustained hypotensive anesthesia to try to control bleeding.
E. Open the pelvic hematoma and apply laparotomy pads with topical hemostatic agents.
Answer: C


DISCUSSION: Pelvic hematomas are not usually the result of common or external iliac artery disruption. Therefore, unless the hematoma is rapidly expanding in the area of one of the major iliac arteries, it is not opened. Instead, the pelvic fracture is fixed externally and arteriography is performed. Usually, bleeding is from smaller arteries as they exit through the fracture site. These are best controlled with embolization. The other options are poor surgical choices that have little chance of success.




65. Which of the following statements about acute arterial occlusion today is/are not true?
A. Most arterial emboli originate in the heart as a result of underlying cardiac disease.
B. It can be treated under local anesthesia.
C. It is usually due to atherosclerotic disease.
D. Surgical treatment can usually be avoided if the lesion is diagnosed early.
Answer: D


DISCUSSION: The majority of arterial emboli originate in the heart as a result of atherosclerotic disease manifested by myocardial infarction, atrial fibrillation, congestive heart failure, or ventricular aneurysm. The incidence of embolization from rheumatic mitral stenosis has decreased significantly. Elderly and debilitated patients who are poor risks for general anesthesia can be treated with the balloon embolectomy catheter technique, which can be performed under local anesthesia via groin incisions. Atherosclerotic disease in the peripheral vessels can cause thrombosis at the site of atheroma. Acute arterial occlusion represents an emergency that is best treated promptly via heparinization and balloon catheter embolectomy.




66. Which of the following statements about lytic agents is/are true?
A. They were first introduced well after the advent of balloon embolectomy.
B. Streptokinase is nonantigenic.
C. Systemic use is the most effective means of delivery.
D. The interval to reperfusion limits their utility in the treatment of advanced ischemia.
Answer: D


DISCUSSION: The balloon catheter technique for embolectomy and thromboembolectomy was introduced in 1963. Lytic agents were undergoing clinical investigation at that time. Streptokinase, derived from streptococci, is antigenic. Urokinase is a naturally occurring enzyme and is not antigenic. Lytic agents have yet to play a major role in the treatment of advanced ischemia in the periphery because of delayed reperfusion, cost, risk of complications, the danger of increased tissue loss.




67. Which of the following is not an indication for postoperative heparinization?
A. Suspected venous thrombosis.
B. Risk of embolism following acute myocardial infarction.
C. Advanced ischemia secondary to acute embolic occlusion.
D. Dissolution of residual thrombus after balloon thromboembolectomy.
Answer: D


DISCUSSION: Conditions that would cause undesirable consequences from rethrombosis (e.g., venous thrombosis, additional embolization from the cardiac source, pre-existing ischemia) are indications for postoperative heparinization. Heparin prevents additional clotting; it is not a thrombolytic agent.




68. Which of the following is/are not true of the embolectomy catheter technique?
A. The balloon should be inflated by the same person who withdraws the catheter.
B. Distal exploration should be carried out in all major branches of the affected extremity.
C. The balloon is designed to dilate as it traverses areas of luminal narrowing.
D. Removal of adherent thrombus requires alternate catheter-based therapy in addition to balloon exploration.
Answer: C


DISCUSSION: The embolectomy balloon should be adjusted in diameter as the catheter is withdrawn to accommodate changes in luminal diameter and to effect appropriate traction. To do so, the operator must simultaneously control the withdrawal and the balloon inflation. The propensity of clot to propagate distally and proximally requires nearby branches to be explored. When it is technically feasible, major branches should be explored. The embolectomy balloon is made of a distensible elastomer that conforms to the intra-arterial surface to maintain wall contact. Dilatation balloons, in contrast, are made of a nondistensible material that can effect dilating force on the arterial narrowing. An increase in the incidence of adherent thrombus from chronic atherosclerotic disease and failed synthetic grafts has promoted the development of more aggressive catheter-based systems. Graft thrombectomy and adherent clot catheters are specifically designed to remove adherent thrombus left behind after balloon exploration.




69. Which of the following is the least reliable indicator of successful thrombectomy?
A. Vigorous back-bleeding after removal of thrombotic material.
B. Arteriographically demonstrated patency of all runoff vessels.
C. Normal distal pulses.
D. Return of normal skin color and temperature.
Answer: A


DISCUSSION: The significant incidence of discontinuous thrombus makes the presence of vigorous back-bleeding an unreliable indicator of distal patency. Proximal material can be cleared, precipitating back-bleeding from all side branches yet still leaving the main vessel occluded more distally. Arteriography, normal distal pulses, and the return of normal skin color and temperature are the best indicators of distal patency, and thus of successful reperfusion. It must be borne in mind that the primary goal of most thromboembolectomy procedures is to return the limb to its preocclusive state. Optimal distal perfusion may require additional therapy for the patient with chronic atherosclerotic disease.




70. Which of the following statements about arteriovenous fistula are correct?
A. The local features characteristic of an arteriovenous communication are demonstrated by the presence of a thrill and bruit with aneurysmal dilatation.
B. An arteriovenous fistula is best managed by ligation of the feeding vessels.
C. The most common type of arteriovenous fistula is iatrogenic, created for vascular access.
D. Branum's or Nicoladoni's sign is increased heart rate when the fistula is compressed.
Answer: AC


DISCUSSION: Local features characteristic of an arteriovenous fistula include a thrill and bruit at the site of the fistula associated with aneurysmal dilatation. Simple ligation of feeding vessels often leads to vascular insufficiency of the peripheral vascular bed secondary to drainage of the flow via collateral circulation. Repair of the artery and vein is considered the optimal therapy. If this is not possible, ligation of the feeding artery and vein proximally and distally can be performed if sufficient collateral circulation has developed, which generally takes months or more after the fistula develops. Branum's or Nicoladoni's sign is a decrease in the heart rate following compression of the fistula.




71. Of the following statements about congenital arteriovenous malformations, which are correct?
A. Patients with complex congenital arteriovenous malformations should as early as possible undergo ligation of feeding vessels.
B. Embolizing large arteriovenous malformations has not been demonstrated to be beneficial.
C. The most common complications of a large arteriovenous fistula are symptoms of congestive heart failure, pain, ulceration, and cosmetic deformity.
D. Most congenital arteriovenous malformations are easily managed with simple excision.
Answer: ABC


DISCUSSION: Complex congenital arteriovenous malformations are often one of the greatest challenges a surgeon may encounter. Management includes accurate diagnosis and determination of the extent of the lesion. The site of communication should be localized by arteriography; however, computed tomography (CT), magnetic resonance imaging (MRI), and duplex Doppler imaging may prove useful in the diagnosis of arteriovenous communications. Ideal surgical management includes closure of the fistula with restoration of arterial and venous continuity. Complex arteriovenous malformations require a multidisciplinary approach, including intra-arterial embolization in conjunction with surgical therapy. Indications for surgery include secondary ischemic complications and congestive heart failure, pain, nonhealing ulcers, and a cosmetic deformity. Ligation of feeding vessels is effective only temporarily, making further treatment, especially embolization, difficult or impossible. Preoperative embolization may allow surgical resection and reduce operative blood loss.




72. Thrombosis occurs frequently in thromboangiitis obliterans (Buerger's disease) in which of the following vessels?
A. Superficial femoral artery.
B. Radial or ulnar artery.
C. Digital arteries.
D. Superficial veins.
Answer: BCD


DISCUSSION: Thromboangiitis obliterans is characterized by thrombosis and inflammation of small and medium-sized peripheral arteries and veins, and by migratory superficial phlebitis. Unlike atherosclerosis, it often involves the upper extremity. Thromboangiitis obliterans rarely involves large vessels such as the brachial or femoral arteries.




73. Which of the following statements about thromboangiitis obliterans (Buerger's disease) are true?
A. The disease affects only young men.
B. The disease is more common in Asia and the Middle East than in the United States.
C. Life expectancy is limited.
D. The usual cause is smoking.
Answer: BD


DISCUSSION: Thromboangiitis obliterans was originally described in young men, who still constitute the overwhelming majority of patients. In recent years, however, several cases have been reported in women, perhaps a reflection of the marked increase in the number of female smokers. For reasons unknown, there is marked geographic variation in the prevalence of thromboangiitis obliterans. The number of cases in the United States has decreased markedly since World War II, but the incidence is much higher in other countries, particularly in the Far East. Although thromboangiitis obliterans often results in peripheral gangrene and may necessitate amputation, long-term life expectancy differs little from that for the general population, unlike that of atherosclerosis. Although rare instances of thromboangiitis obliterans have been reported in nonsmokers, the most common cause is cigarette addiction, which usually starts at an early age. Prolonged remissions often follow cessation of smoking.




74. A 52-year-old man presents with sudden onset of profound cyanosis of the second and third digits of the right hand with gangrene of the tip of the second digit. The remaining digits and the other hand are not affected. Which of the following statements are true?
A. This is characteristic of vasospastic Raynaud's syndrome.
B. Evaluation should include arteriography.
C. A coagulation abnormality may be the cause of this problem.
D. Thoracic sympathectomy is the first-line treatment.
Answer: BC


DISCUSSION: Gangrene is indicative of occlusive disease and not of vasospastic disease. The other factor that makes this unlikely to be vasospasm is the involvement of only two fingers. Because of the possibility of a surgically correctable upstream vascular lesion such as a subclavian aneurysm, patients presenting with embolic-type lesions should have arteriography. Thus this patient, presenting with ischemia of sudden onset in just two digits, should undergo arteriography. Coagulation abnormalities such as antithrombin-III, protein C, or protein S deficiency, or the presence of antiphospholipid antibodies or the lupus anticoagulant, may cause digital gangrene. Conservative therapy—local wound care and débridement—leads to healing in most patients. Sympathectomy should be reserved for patients who fail to heal after months of conservative therapy.




75. A 21-year-old woman presents with digital color changes in response to cold stimulation. Physical examination and laboratory data, including an autoimmune disease screen, are normal. She should be advised that:
A. Her condition is characteristic of vasospastic Raynaud's syndrome and, while she may be at a slightly higher risk for developing a connective tissue disease in the future, there is no evidence of one currently.
B. Her problem with her fingers will get progressively worse and she will eventually lose fingers.
C. She has scleroderma, which will manifest itself at a later date.
D. Her problem is “all in her head.”
Answer: A


DISCUSSION: This presentation is classic for vasospastic Raynaud's syndrome. While these patients are probably at a slightly higher risk for developing a connective tissue disorder in the future, this risk is low. These patients do not invariably progress to tissue loss. This patient has no evidence of scleroderma at the present, and there is no reason to predict that she will develop it. Even though her physical and laboratory examinations are negative, her history is positive for Raynaud's syndrome.




76. Obstructive Raynaud's syndrome can be differentiated from vasospastic Raynaud's syndrome by the:
A. Ice water test.
B. Digital hypothermic challenge test.
C. Antinuclear antibody levels.
D. Digital blood pressure measurement.
Answer: D


DISCUSSION: Patients with obstructive Raynaud's syndrome have fixed palmar and digital arterial lesions with a decrease in digital blood pressure at rest. In contrast, patients with vasospastic Raynaud's syndrome have normal digital arteries at rest and normal digital blood pressure. Therefore the only test that differentiates the two conditions is measurement of digital blood pressure. The ice water and digital hypothermic challenge tests both test for the presence of Raynaud's syndrome, but do not differentiate the spastic from the obstructive type. The presence or absence of antinuclear antibodies does not determine whether obstruction is present, although it is true that many patients with obstructive Raynaud's syndrome will have a positive test.




77. Which of the following statements about upper extremity arterial insufficiency is/are true?
A. Symptomatic ischemia is more common in the upper extremity than in the lower extremity.
B. Vascular injuries from blunt trauma are more common in the upper extremity arteries than in the lower extremity ones.
C. Arteriovenous fistulas frequently follow blunt trauma.
D. The inflammatory process of arteries obstructed by an arteritis should be controlled before a bypass graft is inserted.
Answer: D


DISCUSSION: The larger muscle mass and heavier work loads of the lower extremities may lead to the development of ischemic symptoms in them more often than in the upper extremities. Blunt trauma occurs more often in the lower extremities. Arteriovenous fistulas more frequently follow penetrating trauma than blunt trauma. If the arteritis is not controlled before a bypass graft is placed, the inflammatory process may contribute to early occlusion of the graft.




78. Which of the following statements about upper extremity edema is/are true?
A. Lymphedema is more common than venous edema.
B. Signs and symptoms of venous obstruction include edema, distention of superficial veins, tightness, aching, cyanosis, and pain.
C. Distal venous obstructions are more likely than proximal venous obstructions to cause symptoms in the upper extremity.
D. All patients with symptomatic upper extremity venous thrombosis should receive fibrinolytic therapy.
Answer: B


DISCUSSION: Venous edema is much more common than lymphedema. Any of the signs and symptoms listed may be seen in patients with venous obstruction. The proximal venous obstructions in the axillary, subclavian, and innominate veins are more likely to cause symptoms in the upper extremities than is obstruction of the distal veins. The treatment for symptomatic upper extremity venous thrombosis varies according to the cause of the thrombosis.




79. Which of the following statements about chronic mesenteric ischemia due to atherosclerosis is/are correct?
A. Postprandial pain in these patients is due to gastric hyperacidity and in most cases is relieved with H 2 blockers.
B. Men are more often affected than women.
C. Mesenteric endarterectomy is the surgical treatment of choice, since long-term patency rates are superior to mesenteric bypass.
D. Arteriography is no longer necessary in these cases since noninvasive diagnosis can be established using duplex ultrasound scanning.
E. Surgical treatment is indicated to prevent intestinal infarction in symptomatic patients.
Answer: E


DISCUSSION: The precise cause of postprandial pain in patients with chronic mesenteric ischemia is unknown. Hyperacidity has been observed in some patients with this disease, and gastric pH may be reduced after successful revascularization. No medical therapy, including H 2 blockers, has provided symptomatic relief. Unlike most syndromes of ischemia due to atherosclerosis (coronary, cerebrovascular, peripheral vascular), chronic mesenteric ischemia occurs more frequently in women. The long-term success rates for mesenteric bypass and mesenteric endarterectomy are equivalent; either technique is acceptable. While duplex scanning is a useful noninvasive screening technique in these cases, arteriography is required for definitive diagnosis and to plan revascularization. While prospective, natural history studies have not been done, an increased risk of intestinal infarction is undeniable in these patients. When this occurs, patients rarely survive. Elective surgical revascularization is indicated in symptomatic patients with severe mesenteric arterial occlusive disease.




80. In patients with acute mesenteric ischemia due to mesenteric embolism, which of the following statements is/are correct?
A. Patients often have a history of postprandial pain and weight loss.
B. The use of digitalis may be a predisposing factor to the acute event.
C. Thrombolytic therapy may be attempted in patients without signs of bowel infarction or gastrointestinal bleeding.
D. Arteriography usually reveals the embolus lodged at the orifice of the superior mesenteric artery.
E. At the time of exploration, the proximal jejunum is often viable and ischemia is most severe in the more distal small bowel and colon.
Answer: CE


DISCUSSION: Patients who suffer mesenteric embolism usually have otherwise normal mesenteric arterial anatomy, and ischemic symptoms are acute and profound. A history of chronic gastrointestinal symptoms is most often seen in patients with mesenteric thrombosis. Although cardiac arrhythmias like atrial fibrillation (which predispose to mesenteric emboli) may be treated with digitalis, this has no causative role in mesenteric embolism. Digitalis use has been associated with the development of “nonocclusive” mesenteric ischemia. Thrombolytic therapy with streptokinase or urokinase has been used successfully to treat mesenteric emboli; however, patients with any signs of local or generalized peritonitis should have immediate surgical exploration. Mesenteric emboli usually lodge distally in the main superior mesenteric artery beyond the first jejunal branches and the origin of the middle colic artery. The orifice of the superior mesenteric artery and the proximal branches are normal, which explains the “jejunal sparing” often observed at the time of surgical exploration, even when arteriography has not been performed.




81. Which of the following statements about angiotensin II is correct?
A. It is a decapeptide.
B. It is an enzyme found in high concentration in the pulmonary circulation.
C. It is a direct vasoconstrictor and stimulates aldosterone production.
D. It is a vasoconstrictor and inhibits aldosterone secretion.
Answer: C


DISCUSSION: Angiotensin II is an octapeptide cleaved from angiotensin I by angiotensin-converting enzyme. A direct vasoconstrictor, it also stimulates the adrenal cortical production of aldosterone.




82. Which of the following statements about atherosclerotic obstruction of the renal arteries is true?
A. Lesions are usually short.
B. These lesions are usually found in the distal renal arteries, particularly just beyond branch points.
C. Ostial lesions are best treated with balloon angioplasty.
D. Lesions of this type are the second most common cause of renal artery stenosis.
Answer: A


DISCUSSION: Atherosclerotic renal artery lesions are usually short and found in the proximal renal arteries. Ostial lesions in particular do not respond well to balloon angioplasty, and such lesions are responsible for more than two thirds of renal artery obstructions.




83. Which of the following statements about the treatment of renal artery stenosis is/are true?
A. Though a significant cause of hypertension, renal artery stenosis seldom results in loss of renal function.
B. In patients with medically controlled renovascular hypertension there is no need to consider revascularization.
C. Balloon angioplasty is more effective in patients with atherosclerotic disease as compared with those with fibromuscular disease.
D. In patients with severe atherosclerosis of the aorta, bypass from the splenic or hepatic arteries should be considered.
Answer: D


DISCUSSION: In a significant percentage of patients with renovascular hypertension there is loss of renal mass, which may progress to end-stage renal insufficiency. Ostial lesions respond poorly to balloon angioplasty. If the aorta is severely diseased bypass can be effective using either the hepatic or the splenic artery to supply blood to the kidney.




84. Which of the following statements about venous trauma is/are current?
A. All injured veins can be ligated without any immediate or long-term complications.
B. Attempted repair of injured veins is associated with increased incidence of thrombophlebitis.
C. Attempted repair of injured veins is associated with a high incidence of fatal pulmonary embolism.
D. Careful consideration should be made to repair the injured popliteal vein.
E. Anatomy of the venous system is more variable than anatomy of the arterial system.
Answer: DE


DISCUSSION: Many patients treated by ligation of injured veins suffer no immediate or long-term sequelae. Unfortunately, there are patients who are difficult to identify initially who may have either acute venous hypertension or later suffer chronic venous insufficiency following ligation of large-caliber veins in the lower extremities. The popliteal vein is particularly important to repair if at all possible following injury, as are many of the larger-caliber central veins. Both civilian and military reports have documented that there is not an increased incidence of thrombophlebitis and/or pulmonary embolism associated with attempted repair of injured veins. In contrast to the arterial system, there are many more variations in venous anatomy. This complicates the evaluation of patients with injured veins with examples of a possible bifid popliteal vein and one to five channels comprising the superficial femoral vein.




85. All but one of the following statements is true. Which is not true?
A. Successful clinical repair of injured veins had been effected by the turn of the twentieth century.
B. Initial large experience in managing injured veins came from the battlefields of twentieth-century wars.
C. More than 50% of repaired injured veins thrombose.
D. Phlebography is useful in evaluating variable venous anatomy.
E. Repeated phlebography following attempted venous repair is useful in determining the success rate.
Answer: C


DISCUSSION: Murphy advocated venous repair as early as 1897, after others, including Schede (1882), noted success. Extensive experience in managing large numbers of venous injuries has resulted from major armed conflicts of the twentieth century. Phlebography is particularly valuable for determining venous anatomy and the success or failure of attempted venous repair, and repeated phlebography over the first 72 hours has proven particularly informative. There is a high degree of success in performing lateral suture of lacerations of large-caliber veins as demonstrated by follow-up phlebography.




86. Which of the following statements about injured veins is/are correct?
A. In contrast to the arterial system, it is more difficult to evaluate the patient for suspected venous trauma.
B. There is no simple method similar to palpating distal pulses following arterial repair to evaluate the status of attempted venous repair.
C. Recanalization of initial thrombosis of attempted venous repair is more common than in the arterial system.
D. Doppler ultrasound—and more recently color-flow duplex—have been increasingly helpful in evaluating integrity of the venous system.
E. Prevention of venous stasis is important in the immediate postoperative period after attempted venous repair.
Answer: ABCDE


DISCUSSION: There are significant differences between the arterial system and the lower-pressure venous system. Presence or absence of distal pulses associated with possible arterial injuries provides a simple mechanism for evaluating patients with potential injury, while there is no simple method of evaluating patients for potential venous injury. Doppler ultrasound—and more recently color-flow duplex—have been increasingly helpful in evaluating the integrity of the venous system. If thrombosis of an attempted venous repair occurs, there is a high probability that recanalization of the thrombus will occur. This is in contrast to the arterial system, where recanalization of thrombosis of an attempted arterial repair is a rare occurrence. Venous stasis must be prevented in the immediate postoperative period following attempted venous repair to reduce the possibility of thrombosis.




87. Concerning the normal structure of blood vessels, the following is/are true:


a. In utero, hemangioblasts give rise to both vascular conduits and hematopoietic tissue
b. In development, smooth muscle tubes precede endothelium
c. After birth, growth of large vessels does not change the number of elastic and smooth muscle layers
d. Adventitia includes the external elastic lamina
Answer: a, c


The earliest vascular primordia in the embryo are isolated hemangioblasts that give rise to both vascular conduits and hematopoietic tissue. Endothelial cells organize at sites of vessel development followed by mesenchymal cells that form the outer layers. The number of elastic and smooth muscle layers remains constant after birth, although wall mass increases due to proliferation. Adventitia lies outside the external elastic lamina




88. Among the theories of atherosclerosis, the following is/are true:


a. Fatty streaks in the aorta of children do not predict atherosclerosis or heart attacks
b. Aging induces non-atherosclerotic thickening of the intima
c. T-lymphocytes are present in atheromas
d. The reaction-to-injury hypothesis serves to explain the characteristic lipid accumulation
Answer: a, b, c


It is true that fatty streaks in the aorta and coronary arteries of children are found in populations without increased incidence of atherosclerosis or heart attacks. Similarly, aging induces gradual thickening of the intima throughout the arterial tree which is not atherosclerotic. A variety of leukocytes including T-lymphocytes are present in atheromas. The reaction-to-injury hypothesis explains smooth muscle growth in atherogenesis but fails to provide an explanation for lipid accumulation or the monoclonal nature of the atherosclerotic plaque.




89. Concerning in-vivo regulation of the anticoagulated state by endothelium, the following is/are true:


a. Heparan-ATIII inactivates only thrombin
b. Thrombomodulin serves only to bind thrombin
c. Production of von Willebrand factor (VWF) inactivates platelets
d. Endothelial cells can secrete tissue factor
Answer: d


Endothelium synthesizes heparan which, like heparin, increases the affinity of ATIII for thrombin which is inactivated along with other serine proteases, including factors VII, IX and X. Thrombomodulin, in addition to binding thrombin, activates protein C which binds with protein S to inactivate factor Va. On the procoagulant side, endothelial cells produce VWF which binds platelets and are capable of secreting tissue factor.




90. Concerning medial and intimal thickening, the following is/are true:


a. Increase of wall mass is a consequence primarily of smooth muscle cell proliferation
b. Smooth muscle cells are normally quiescent at maturity
c. Transplanting a vein into the arterial circuit causes both endothelial and smooth muscle proliferation
d. Heparin can suppress both proliferation and migration of smooth muscle cells
Answer: a, b, d


Endothelial proliferation does not contribute to an increase in wall mass which is secondary to smooth muscle cell proliferation. Smooth muscle cells are normally quiescent at maturity and their proliferation and migration are inhibited by heparin. Transplanting a vein into the arterial circuit causes some endothelial cell loss and smooth muscle cell proliferation.




91. Concerning regulation of arterial luminal area, the following is/are true:


a. The major determinant of arterial diameter is blood pressure
b. Compensatory vasodilation occurs until more than 40% of area inside the internal elastic lamina is obstructed in coronary arteries
c. Vasodilating nitric oxide is derived from adenosine
d. When endothelium is absent, thrombin causes vasoconstriction.
Answer: b, d


The major determinant of arterial diameter is blood velocity as demonstrated by post-stenotic dilation. This adaptation by wall relaxation is limited to 40% of the area inside the internal elastic lamina in coronary arteries. The predominant vasodilator, nitric oxide, is derived from arginine. When endothelium is absent, a number of vasodilators, including thrombin, produce vasoconstriction.




92. Concerning regulation of smooth muscle cell growth, the following is/are true:


a. Serum derived from plasma has substantially more growth promoting activity than serum from whole blood.
b. Fibroblast growth factor (basic) is responsible for the first wave of proliferation in experimental arterial injury
c. The gene for platelet derived growth factor (PDGF) is nearly identical to the oncogene v-sis
d. Sympathectomy promotes the increase in DNA in the media of developing arteries and in hypertension
Answer: b, c


The observation that serum derived from whole blood has substantially more growth promoting activity than serum from plasma led to the discovery of PDGF. Basic FGF is responsible for the first wave of proliferation in experimental carotid artery injury. The gene for PDGF is nearly identical to the oncogene v-sis raising the possibility that wound healing and malignant growth might have similarities of regulation. Sympathectomy inhibits the increase in DNA in the media of developing arteries and in hypertension.




93. A 21-year-old man with premature arteriosclerosis and mental retardation is found to have homocystinuria. The following is/are true:


a. Presence of mental retardation is atypical for homocystinemia
b. The specific enzyme deficiency responsible is homocysteine methyl transferase
c. Arteriosclerotic plaques in this condition are atypically void of lipid deposition
d. Homocysteine exists in plasma in three forms: protein bound, mixed and free
Answer: c, d


Homocystinuria reflects homocystinemia which is associated with ectopia lentis, mental retardation and thromboembolic disorders as well as arteriosclerosis. There are three enzyme deficiencies known to cause the disorder as well as deficiencies of the cofactors pyridoxine, cobalamin and folate. Lipid deposition in plaques is characteristically absent. Homocysteine exists in plasma as the mixed disulfide homocysteine cysteine, as free and as protein bound homocysteine.




94. A 22-year-old male basketball player with back pain is found to have a dissecting aortic aneurysm. the follow is/are true:


a. In Marfan’s syndrome, a disorder of type I collagen underlies the observed cystic medial necrosis
b. In type IV Ehlers-Danlos syndrome, little or no type III collagen is produced and arterial rupture is likely
c. In pseudoxanthoma elasticum, the medial elastic fibers are replaced by xanthoma cells which calcify
d. In arteria magna syndrome, the media is devoid of elastic tissue and coronary artery disease is common
Answer: a, b, d


Cystic medial necrosis is associated with aortic dissection at an early age and can be due to Marfan’s syndrome with its disorder of type I collagen or type IV Ehlers-Danlos where little or no type III collagen is produced. In pseudoxanthoma elasticum, the medial elastic tissue is replaced by calcific deposits and there are xanthoma-like cutaneous papules. In arteria magna syndrome, elastic tissue is absent in the media and associated coronary artery disease is common.




95. A 38-year-old male smoker with gangrenous changes in the toes of both feet has an arteriogram showing normal vessels to the popliteal trifurcation and multiple occlusions distally in small vessels. The following is/are true:


a. Hyperlipidemia, diabetes, and autoimmune disease must be ruled out to make the diagnosis of Buerger’s disease
b. Plethysmographic evidence of digital obstruction in all four extremities with normal proximal vessels is sufficient evidence for Buerger’s disease without arteriography
c. The most important treatment for Buerger’s disease is regional surgical sympathectomy
d. In contrast to the lower extremities, Buerger’s involvement of the upper extremities rarely leads to amputations
Answer: a, b, d


Buerger’s disease is a panarteritis associated with intraluminal thrombus in young male smokers. Diabetes, hyperlipidemia and autoimmune diseases must be ruled out to fulfill the diagnostic criteria, but the diagnosis can be made plethysmographically by evidence of small vessel obstruction in all four extremities. Cessation of all tobacco use is the most important treatment. Management is conservative with only rare limb loss in the upper extremities as opposed to the lower.




96. A 42-year-old Asian woman with a history of recurrent deep venous thrombosis presents with a pulsatile mass in the abdomen confirmed on ultrasound to be an abdominal aortic aneurysm. The following is/are true:


a. History and findings suggest Kawasaki disease
b. History and findings suggest polyarteritis nodosa
c. Venous thrombosis is more common than arterial disease in these patients and the presence of an aneurysm portends a high mortality rate
d. Replacement of an aneurysm with a graft in Behcet’s disease is associated with recurrent aneurysms and thrombosis
Answer: c, d


Kawasaki disease is a disorder of infants and children with coronary aneurysms. Polyarteritis nodosa usually occurs in males and the inflammatory process involves small and medium-sized muscular arteries. Behcet’s disease is a vasculitis that produces venous thrombosis, and when arterial aneurysms are present, the mortality rate approaches 20%. Because of the fragility of the arteries, recurrent aneurysm formation is likely.




97. A 32-year-old woman with severe hypertension is found to have renal artery changes as shown in Figue 69-1. The following statement/s/ is/are true:


a. Next to the renal artery, this process affects the carotid and coronary arteries most commonly
b. In the most common variant of this disorder, the media is infiltrated with increased collagen, fibrous connective tissue and glycosaminoglycans
c. If similar disease is found in the carotid, it should be treated, even if asymptomatic
d. Appropriate treatment includes percutaneous transluminal balloon angioplasty
Answer: b, d


Fibromuscular dysplasia is an abnormality of unknown etiology primarily affecting women (90%) and in the renal arteries. The carotid and iliac arteries are the next most frequently affected. Medial fibroplasia is the most common pathology with the pathological findings in answer b. Surgical treatment is indicated only for symptomatic stenoses since many asymptomatic cases exist for which the natural history is unknown. In addition to surgical procedures, balloon angioplasty of main renal artery lesions is acceptable treatment.




98. Ten years after irradiation of the neck for a tonsillar carcinoma, a 59-year-old woman is found to have symptomatic carotid artery disease. Arteriogram shows a 70% irregular stenotic lesion. The following is/are true:


a. Replacement of the artery should be planned due to radiation induced arterial injury
b. The pathology is most likely to be an inflammatory reaction with endothelial sloughing and thrombosis
c. If atherosclerotic disease is found, the plaque will be no different than nonirradiated plaques
d. The patient should be managed medically because of the radiation arterial injury
Answer: c


Radiation-induced arterial injury produces three types of injury, the earliest post-treatment consisting of inflammatory reaction with endothelial slough and thrombosis. Later, there may be fibrotic changes in the wall producing stenosis or accelerated atherosclerosis. The latter lends itself to standard endarterectomy and the plaque is indistinguishable from non-irradiated plaque.




99. A 23-year-old woman with fever, myalgia and anorexia presents with hypertension and a cool, ischemic left arm. Angiography shows multiple stenoses of the subclavian and renal arteries. The following is/are true:


a. Coronary angiography is indicated with high likelihood of finding coronary disease
b. Endarterectomy of the lesions would be preferred to transluminal angioplasty
c. The presentation is more suggestive of Behcet’s disease than Takayasu arteritis
d. Preferred management consists of corticosteroids
Answer: d


The presentation is most suggestive of Takayasu arteritis which tends not to involve the coronary arteries. A variety of operations have been used in these patients but endarterectomy is not recommended because of a high incidence of early failure. The preferred management is corticosterioids.




100. A 58-year-old woman presents with a history of severe headache, visual field loss and a transient myalgia involving the back and shoulders. The following is/are true:


a. A tender, nodular temporal artery would indicate a picture compatible with temporal arteritis
b. The presentation is most compatible with giant cell arteritis
c. Steroids should be avoided if an operation is planned
Angiography is most likely to show irregular surface stenosis
Answer: a, b


The presentation is typical for a patient with temporal arteritis, which is a form of systemic giant cell arteritis. It is characterized by chronic inflammation of the aorta and its major branches. Corticosteroid therapy is indicated because of its success in relieving symptoms whether or not an operation is planned. Angiographic findings in this condition show smooth rather than irregular surface stenoses.




101. Concerning the fibrinolytic system the following is/are true:


a. Plasminogen is an a-globulin
b. Fibrin but not fibrinogen is lysed by plasmin
c. The main inhibitor of plasmin is a2-macroglobulin
d. TAP is activated during fibrin bonding to plasminogen
Answer: d


Plasminogen is a b-globulin that is converted to plasmin by a number of activators. Plasmin lyses and destroys both fibrin and fibrinogen. The main inhibitor of plasmin is a2-antiplasmin; a2-macroglobulin, ATIII and antitrypsin are less important inhibitors. During fibrin formation, both plasminogen and TAP bind to it specifically, and TAP is activated.




102. Concerning platelet function in vascular disease, the following is/are true:


a. Platelet aggregation is the initial step in thrombogenesis when subendothelial structures are exposed.
b. Non-homogeneous distribution of platelets towards the vessel wall is enhanced at increasing shear rates
c. The platelet glycoprotein (GP) which is the principal collagen receptor is GPIb-IX
d. Fibrinogen binding to GPIIb-IIIa is a prerequisite for all platelet aggregation
Answer: b, d


Platelet adhesion is the initial step in thrombogenesis when subendothelial structures are exposed. Platelet adhesion and their non-homogeneous distribution toward the vessel wall are enhanced at increasing shear rates. The platelet GP which is the principal collagen receptor is GPIa-IIa, GPIb-IX is the one which binds to von Willebrand factor. Fibrinogen binding to GPIIb-IIIa is a prerequisite for all platelet aggregation.




103. Concerning the inhibition of intravascular coagulation, the following is/are true:


a. Heparin accelerates the effects of ATIII to a greater extent than native heparin sulfate
b. Protein C but not protein S is vitamin K dependent
c. Protein Ca stimulates the release of TAP from endothelial cells
d. ATIII neutralizes factors Xa, IXa and IIa
Answer: a, c, d


The rate of inhibition of activated coagulation factors by ATIII is dramatically accelerated by heparin, and to a lesser extent by heparin sulfate. Both protein C and protein S are vitamin K dependent, and protein Ca does stimulate the release of TAP from endothelial cells. ATIII neutralizes factors Xa, IXa and IIa (thrombin).




104. Concerning hypercoagulable syndromes, the following is/are true:


a. Acquired hypercoagulable states are more common than congenital disorders
b. Fatal neonatal thrombosis is associated with severe dysfibrinogenemia
c. Heparin associated thrombocytopenia is due to an antibody that attaches to the platelet Fc receptor
d. The lupus anticoagulant induces a hemorrhagic diathesis
Answer: a, c


Acquired hypercoagulable states are much more common than congenital disorders at our present level of understanding. Fatal neonatal thrombosis is associated with homozygous ATIII deficiency. Heparin associated thrombocytopenia is associated with an IgG antibody that attaches at the Fc receptor and triggers platelet secretion and aggregation. The lupus anticoagulant is an antiphospholipid antibody that induces a thrombotic rather than a hemorrhagic disorder.




105. Concerning the treatment of thrombotic vascular disease, the following is/are true:


a. Aspirin is successful in preventing venous as arterial thromboembolism
b. Dipyridamole enhances the ability of aspirin to prevent arterial thrombosis
c. Ticlopidine is more effective than aspirin in patients with cerebrovascular disease
d. Ticlopidine prevents fibrinogen binding to the GPIIb-IIIa receptor complex
Answer: c, d


Aspirin and other anti-platelet drugs are effective in preventing thrombosis in arterial but not venous thrombotic disorders. Dipyridamole has been found to be ineffective in rigorous clinical trials. Ticlopidine is more effective than aspirin in patients with cerebrovascular disease, and serves to prevent fibrinogen binding to the platelet GPIIb-IIIa complex.




106. An 82-year-old man with a long history of coronary and peripheral vascular disease presents with an acutely ischemic right lower extremity. The following is/are true:


a. The first step in management should be an arteriogram
b. If intractable congestive heart failure is present, non-operative treatment with heparin would be appropriate
c. If prolonged ischemia has occurred, reperfusion should be accompanied by sodium bicarbonate
d. Regardless of the period of ischemia, fasciotomy should be based on the findings postoperative
Answer: b


The first step in the management of acute limb ischemia in any patient is heparin anticoagulation. If intractable heart failure is present, heparin treatment would be appropriate without operation. If prolonged ischemia has occurred, the venting of the first 3–500 ml of venous outflow will allow conservation of RBCs and avoidance of the consequences of high levels of potassium. If the duration of ischemic has exceeded 4 hours, a 4 compartment fasciotomy should be performed at the time of restoration of perfusion.




107. Two days following coronary angiography and angioplasty, a 47-year-old male diabetic develops painful blue toes on both feet. The following is/are true:


a. It is very unlikely that there is any connection between the catheterization and the extremity problem
b. The appropriate treatment is vasodilators and an antiplatelet agent
c. If both superficial femoral arteries are obstructed, the most likely etiology is in-situ microvascular thrombosis
d. If renal failure or pancreatitis develops, the outlook for long term survival is very poor
Answer: d


Artheroembolism results from plaque rupture or manipulation at catheterization and is much more frequent after catheterization than suspected clinically. Since repetitive events and additional complications are expected, prompt arteriography should be performed to delineate the possible site of origin which is then excised, endarterectomized or bypassed as the only effective treatment. Since plaque debris is very small, it can readily pass through collateral vessels to lodge in arterioles, and major vascular occlusion is no barrier. The kidney is the most common organ affected and if renal failure or pancreatitis develops as a sign of generalized atheroembolism, the outlook is poor with life expectancy measured in months.




108. A 39-year-old woman with embolic occlusion of an iliac artery is subject to an operating room delay before perfusion can be restored. The following is/are true:


a. Ischemia for longer than 3 hours will result in muscle fiber autolysis
b. Earliest ultrastructural changes of ischemia in muscle include mitochondrial swelling and loss of glycogen granules
c. Phosphocreatine mediated rephosphorylation of ADP occurs for about 3 hours after ischemia
d. Capillary thrombosis is the most likely explanation for the “no-reflow” phenomenon
Answer: b, c


Skeletal muscle can tolerate ischemia by anaerobic glycolysis for up to 6 hours. The earliest ultrastructural changes in ischemic muscle include mitochondrial swelling and loss of glycogen granules. During ischemia, ATP levels are maintained by phosphocreatine mediated rephosphorylation of ADP until phosphocreatine levels are exhausted after about 3 hours. Of the possible causes of the “no-reflow” phenomenon, capillary obstruction by leukocytes is more likely than capillary thrombosis.




109. A 70-year-old man presents with sudden pain and ischemic changes in his left leg. An arterial embolus is suspected. The following is/are true:


a. The most likely source of an arterial embolus is from intracardiac thrombus on a previous MI
b. If atrial fibrillation (AF) is present, it is known that chronic AF is less likely to produce embolism than paroxysmal AF
c. Currently, the most common cause of AF is ischemic rather than rheumatic heart disease
d. Aspirin is more effective than coumadin in AF for reducing risk of stroke and cardiovascular mortality
Answer: c


Approximately 80–90% of arterial emboli originate in the heart, and 2/3 are secondary to AF. Chronic AF carries an annual risk of 3–6% of significant embolic complications while paroxysmal AF has a lower risk. Traditionally, rheumatic heart disease was the most common cause of chronic AF, but with its decline, ischemic heart disease has become the most common cause. Drug therapy in AF will reduce the risk of stroke, but aspirin is less effective in this regard than coumadin.




110. In discussing risk and outcomes of the patient in the previous question, the following is/are true:


a. If renal failure occurs, the mortality rate is about 50%
b. If arterial embolism is confirmed, the patient should receive lifelong anticoagulation
c. Postoperative amputation is unlikely if the embolectomy is successful
d. Postoperative death from pulmonary embolism is unlikely
Answer: a, b


There has been only modest improvement in the mortality and morbidity after arterial embolectomy in the past 40 years, and if renal failure occurs, the mortality rate is about 50%. Recurrence of arterial embolism without anticoagulation occurs in 28–45% of patients and justifies prolonged anticoagulation which reduces the incidence of recurrent embolism. In addition to a high postoperative mortality rate, amputations are required in about 15% of patients. Pulmonary embolism is the 2nd most common cause of death after embolectomy, reflecting the incidence of DVT in 7–27% of patients after arterial embolectomy.




111. The clinical manifestations of the patient in the previous question would include:


a. Loss of sensation to deep pain as one of the earliest signs
b. Paresthesia would be noted in a classical dermatome distribution
c. Early pallor is due to both diminished skin blood flow and reflex vasoconstriction
d. Involuntary muscle contraction indicates that restored flow cannot save the extremity
Answer: c, d


The earliest limb changes in ischemia are in sensory nerves with small nerve fibers having increased sensitivity resulting in loss of sensation to light touch. Sensation to deep pain, pressure and temperature are preserved until late. The paresthesias that occur are in a glove or stocking-like distribution rather than by dermatome. Early pallor is due to decreased blood flow as well as reflex vasoconstriction. Among signs of irreversible limb ischemia are complete anesthesia and involuntary muscle contraction.




112. A 51-year-old man with a history of transmural MI one month ago presents with sudden occlusion of his abdominal aorta. The following is/are true:


a. Most likely location of the MI is anterolateral
b. The vast majority of emboli occur within 6 weeks of the occurrence of the MI
c. Occurrence of arterial embolism does not affect the overall mortality
d. Heparin can reduce the incidence of embolism after MI
Answer: a, b, d


Acute MI with endocardial thrombus is the second most common cause of arterial embolism most commonly within 6 weeks, and the most typical location postmortem is anterolateral. Arterial embolism after MI is associated with an increase in mortality rate. Heparin following acute MI has been shown to reduce the incidence of systemic arterial embolism.




113. A 67-year-old man with acute popliteal arterial embolism has a negative cardiac echo for source of the thrombus. The following is/are true:


a. Most likely non-cardiac source is a thoracic aortic aneurysm
b. Embolism is more common from femoral than popliteal arterial aneurysms
c. Emboli from popliteal aneurysms are often clinically silent
d. Embolism is rare from subclavian artery aneurysms
Answer: c


The most likely non-cardiac source of an arterial embolism is an infrarenal abdominal aortic aneurysm. Arterial embolism is more frequent from popliteal than femoral aneurysms, and these embolic events are often clinically silent. Subclavian artery aneurysms give rise to peripheral embolism in up to 33% of patients.




114. In regards to the previous case, the following statement(s) concerning the distribution of arterial emboli is/are true:


a. Change in arterial diameter is a more important determinant of embolic site than flow rate
b. Aortic valvular disease is more often associated with cerebral embolism than mitral valve disease
c. Among embolic sites, renal emboli are least detected clinically
d. The most common site for an arterial embolus is the aortic bifurcation
Answer: a, c


Most arterial emboli lodge at bifurcations, where there is a sudden change in arterial diameter. Flow rate does not correlate with sites of embolism. It is mitral valve disease with associated atrial fibrillation that is most frequently associated with cerebral embolism. The discrepancy between clinical and autopsy evidence of embolism is significant for renal emboli where the clinical diagnosis is made in less than 1% of cases. The most common site for an arterial embolus is the common femoral artery.




115. Concerning cellular metabolism, the following is/are true:


a. Anaerobic metabolism is about half as efficient in energy production as normoxic metabolism.
b. Loss of cellular Ca++ as ion pumping fails activates phospholipase
c. Cellular swelling serves to protect the cell
d. Ketone as an alternative energy source has been shown to be beneficial in ischemia
Answer: b, d


The shift from normoxic to aerobic metabolism results in a dramatic loss of efficiency from 38 mol ATP to a net of 2 mol per molecule of glucose, a 94% reduction. With loss of ion pumping, free Ca++ accumulates and triggers phospholipase activation. Cellular swelling and interstitial edema increase diffusion distances further compromising oxygen and substrate delivery. In a number of experimental conditions, ketone as an alternative to glucose has proved beneficial.




116. A 62-year-old woman with embolic femoral artery occlusion is facing a delay before circulation can be restored. To minimize ischemic injury, the following is/are true:


a. Increased oxygen saturation is beneficial
b. Injury will continue during reperfusion
c. Low collateral flow is more harmful than no flow
d. Concern regarding reperfusion injury should not delay revascularization
Answer: b, c, d


A significant part of ischemic injury occurs during reperfusion from generation of oxygen free radicals, but this should not delay efforts to provide revascularization. Excess oxygen levels are harmful rather than helpful and low collateral flow has been demonstrated experimentally to be more harmful than no flow.




117. Concerning tolerance to tissue ischemia, the following factor(s) are important variables determining organ failure:


a. Resting metabolic rate
b. Anaerobic glycolysis
c. Autonomic nerve supply
d. Efficiency of existing collaterals
Answer: a ,b, d


Although the specific tolerance of various tissues and organs to ischemic injury is variable, the resting metabolic rate, anaerobic glycolysis and efficiency of existing collaterals are important variables. Autonomic nerve supply does not play a major role.




118. Concerning ischemic cellular injury, the following is/are true:


a. Prolonged hypoxic metabolism is inefficient but not harmful to cells
b. Microvascular endothelium is a significant source of xanthine oxidase
c. The “no-reflow” phenomenon is due to arteriolar spasm
d. Endothelial cells lack the defense system for oxidative damage
Answer: b, d


Prolonged hypoxic metabolism is not only inefficient but the accumulation of lactic acid produces direct injury to organelles, alters enzyme activity, and enhances cytokine production. The microvascular endothelium reacts to ischemia by production of xanthine oxidase, but lacks the defense system for oxidative damage. The “no-reflow” phenomenon is associated with an increased number of WBCs adherent to the luminal surface of the microcirculation and cellular swelling.




119. Concerning remote effects of localized ischemia, the following is/are true:


a. Lower torso and limb ischemia induce pulmonary injury via TXA2 and WBC effects
b. ATP degradation produces adenosine which induces systemic hypertension
c. Adenosine also causes renal and pulmonary vasoconstriction
d. Oxygen radicals are continuously produced by normal metabolic processes
Answer: a, c, d


Both experimental and clinical studies have linked WBC dependent pulmonary injury mediated by TXA2. Adenosine release from ATP degradation contributes to systemic hypotension since it is a vasodilator in most vascular beds but a vasoconstrictor in the renal and pulmonary vasculature. Oxygen radicals are continuously produced by normal metabolic processes and handled by quenching and scavenging systems.




120. Which of the following is/are true regarding exercise testing and reactive hyperemia in patients with peripheral vascular occlusive disease?


a. Normal individuals walk on a treadmill at 2 m.p.h. at a 10% grade without experiencing leg pain and ankle pressure remains unchanged after exercise
b. In patients with arterial obstruction, pain usually forces cessation of walking after 2–3 minutes and the ankle pressure measured immediately after exercise is diminished
c. The time required for pressure to return to baseline is usually 2–3 minutes
d. Reactive hyperemia may be used as a substitute for treadmill exercise
Answer: a, b, d


Normal subjects walk without pain and do not drop their ankle pressure after exercise. In patients with arterial obstruction there is a drop in ankle pressure after exercise and the severity of that drop is roughly proportional to the severity of the occlusive process. Likewise the time for pressure to return to pre-exercise levels is proportional to the severity of the occlusive process and may exceed 20 minutes in severely diseased extremities. The reactive hyperemia test is quite sensitive in patients who cannot exercise, and may be used as a substitute for treadmill exercise.




121. Which of the following is/are true with respect to transcutaneous PO2 (TcPO2) measurements?


a. TcPO2 levels provide an index of the adequacy of tissue perfusion and depends on the quantity of oxygen delivered and that extracted to meet metabolic demands
b. Extremity TcPO2levels are typically normalized to a well perfused area, such as the infraclavicular skin
c. TcPO2levels average about 60 mmHg in normal limbs
d. Patients with limb threatening ischemia usually have values less than 20 mmHg and may approach 0
Answer: a, b, c, d


Transcutaneous PO2 levels provide an index of the adequacy of tissue perfusion. They depend on the quantity of oxygen delivered by the blood and that extracted to meet metabolic demands. Because oxygen supply is a function of the arterial PO2, cardiac output and age peripheral measurements must be compared with levels from a well perfused central area, such as the infraclavicular skin. In normal limbs transcutaneous oxygen levels average 60 mmHg or 90% of the infraclavicular value. Many claudicants have resting values in the normal range, whereas in patients with limb threatened ischemia the values are usually less than 20 mmHg and many be zero.




122. Which of the following is/are appropriate candidates for exercise testing?


a. The patient with symptoms of intermittent claudication but normal resting ankle brachial indices
b. The patient with rest pain, nonhealing ulcers or gangrene
c. If the resting ankle pressure is below 30–40 mmHg
d. The patient with blue toe syndrome and readily palpable pedal pulses
Answer: a


When the patient’s presenting complaints are compatible with claudication and the ABI is normal or nearly so treadmill exercise is quite helpful in unmasking significant arterial occlusive disease. In patients with obvious rest pain, nonhealing ulcers or gangrene the diagnosis of significant peripheral vascular disease is obvious, likewise for the patient’s whose resting ankle blood pressure is below 30 or 40 mmHg. Finally, in patients with atheroemboli and palpable pedal pulses, toe brachial indices may be helpful but exercise testing is not. Search for the embologenic source is more appropriate.




123. Which of the following statements is/are true regarding the use of duplex scanning as a means to follow and monitor bypass grafts?


a. Duplex scanning is accurate and cost effective
b. A localized increase in systolic velocity greater than 25% compared to adjacent segments in the graft identifies a diameter reduction of at least 50%
c. Peak systolic velocities should be less than 40 cm/sec throughout the graft
d. Arterial venous fistulas associated with in situ bypass grafts are difficult to detect with a duplex scanner
Answer: a


Duplex scanning is an excellent way to monitor bypass grafts. It can detect graft threatening defects before the patient becomes symptomatic and before the ankle pressure begins to drop. A localized increase in systolic velocity greater than 100% of that of the adjacent graft identifies a diameter reduction of more than 50%. Peak systolic velocities less than 40 cm/sec are an ominous sign of markedly reduced flow. Arterial venous fistulas are regularly recognized by their pattern of localized flow disturbances with increased velocities at the site of the fistula and immediately proximal to the fistula and concomitant decreased velocities just below the fistula.




124. Which of the following is/are true with respect to ankle blood pressure and ankle brachial index (ABI)?


a. An ABI of less than 0.92 almost always indicates hemodynamically significant arterial disease
b. Claudicants have a wide range of ABIs with average values of 0.6 +/– 0.15
c. In limbs with rest pain the mean ABI is typically 0.25 +/– 0.13
d. In limbs with impending gangrene ABIs seldom exceed 0.25 and average about 0.05 +/– 0.08
Answer: a, b, c, d


The normal person’s resting ankle brachial blood pressure usually exceeds brachial blood pressure. In persons with stenotic lesions that do not reduce the diameter of the arterial lumen by more than 50% there may be no change in resting ankle brachial blood pressure. Absolute ankle pressures of less than 40 mmHg always indicate severe arterial compromise regardless of the ABI. The ranges given above have been empirically derived from assessing large numbers of patients. When the arteries are incompressable secondary to calcification spuriously high ankle pressures may be obtained.




125. Which of the following is/are true regarding Doppler assessment?


a. Conventional Dopplers admit an ultrasonic beam in the frequency of 2–10 MHz
b. The sound frequency changes in inverse proportion to the velocity of the moving particles (red blood cells) and the cosign of the angle of insonnation
c. The frequency shift is not audible
d. More information can be obtained by spectral analysis
Answer: a, d


Of all of the diagnostic methods used in the noninvasive lab, Doppler ultrasound has the most utility. In clinical usage Doppler instruments emits an ultrasonic beam at 2–10 MHz. The frequency of the sound is changed in proportion to the velocity of moving particles (red blood cells) and the cosign of the angle of insonnation that the beam makes with the velocity vector. The frequency shift is in the audible range and listening with a pocket Doppler provides a quick and simple method of assessing blood flow. Spectral analysis with frequency on the vertical axis, time on the horizontal axis and amplitude changes indicated by an increasing intensity on the grey scale provides considerably more information, particularly with respect to flow disturbance which produce a broadening of the normally narrow bands of frequencies which parallel the flow envelope.




126. Which of the following is/are true with respect to assessment of the carotid circulation?


a. The external carotid artery flow pattern resembles those obtained from peripheral arteries
b. The internal carotid artery maintains forward flow throughout the cardiac cycle
c. Peak systolic velocity exceeding 200 cm/sec suggests a stenosis greater than 50%
d. An end diastolic velocity greater than 120 cm/sec suggests a stenosis greater than 80%
Answer: a, b, d


The brain is a low resistance vascular bed and internal carotid artery blood flow is positive throughout the cardiac cycle. The external carotid artery which supplies the muscles of the face and neck resembles peripheral arteries. A great deal of attention has focused on determining criteria for estimating stenosis severity. A peak systolic velocity exceeding 130 cm/sec suggests a stenosis of greater than 50% and end diastolic velocity greater than 120 cm/sec suggests a stenosis greater than 80%. An internal carotid/common carotid peak systolic velocity ratio of 4.0 or more and an end diastolic velocity of 100 cm/sec have been proposed as criteria for identifying diameter stenosis of 70%.




127. Which of the following is/are true regarding the assessment of renal artery obstruction with duplex scanning?


a. There is no flow reversal in early diastole in the renal artery
b. Renal artery to aortic peak systolic velocity ratios that exceed 3.5 indicates the presence of a 60% diameter stenosis
c. Duplex scanning regularly identifies accessory renal arteries
d. Duplex scanning cannot be recommended as a means for monitoring renal artery reconstruction
Answer: a, b


The kidneys represent a low resistance vascular bed, therefore flow in renal arteries is positive throughout the cardiac cycle and there is no flow reversal in early diastole as is seen during the assessment of peripheral arteries. A ratio of renal artery to aortic peak systolic flow that exceeds 3.5 indicates the presence of a 60% diameter stenosis. Sensitivities greater than 80% and specificities greater than 90% have been reported using these criteria to predict significant stenoses of main renal arteries. Unfortunately duplex scanning often fails to detect accessory renal arteries or segmental branch disease. It is quite reasonable as a screening technique for patients with suspected renal artery hypertension and as an accurate method for monitoring renal artery reconstruction.




128. Which of the following is/are true regarding normal peripheral arterial flow waves?


a. Flow is antegrade and rapidly accelerated in early systole
b. There is a rapid deceleration phase during which velocities fall to 0
c. A short period of flow reversal occurs in early diastole
d. Low level forward flow continues throughout the remainder of diastole
Answer: a, b, c, d


The characteristic triphasic audible Doppler signal represents distinct phases: rapid acceleration in early systole, a sharp peak at maximal velocity, a rapid deceleration phase, a short flow reversal phase in early diastole secondary to elastic recoil and finally low level forward flow throughout the remainder of diastole. Beyond an obstruction the flow pulse becomes more rounded, the acceleration phase is less rapid, the peak less well defined, the reverse flow component disappears, and the velocities remain above baseline throughout diastole.






129. Which of the following is/are true with regard to diabetes mellitus as a risk factor for atherosclerosis?


a. Atherosclerosis is the cause of death in approximately 75% of diabetic persons
b. Following a myocardial infarction diabetic persons have a higher rate of in-hospital mortality and a higher five year mortality than nondiabetic persons
c. The length of time one has diabetes appears to be a factor for the development of atherosclerosis whereas the severity of the diabetes appears to have little relationship to the development of vascular disease
d. The impact of diabetes on the development of vascular disease appears greater in women than in men
Answer: a, b, c, d


Atherosclerosis is the most common complication of diabetes developing in almost 85% of the patients who survive more than 20 years after the diagnosis. Atherosclerotic complications are the cause of death in 75% of diabetic persons; in the general population artherosclerotic complications account for about one-third of all deaths. Diabetic persons fare less well after a myocardial infarction with significant increases in-hospital mortality and decreases in five year survival. The impact of diabetes on cardiovascular complications seems to be proportionate to the duration of the diabetes rather than the severity of the diabetes and women diabetics fare markedly less well than men with diabetes. In men, diabetes is the least important of the major risk factors whereas in women the impact of diabetes exceeds that of cigarette smoking.




130. Which of the following statement(s) is/are correct with regard to Type IV hyperlipoproteinemia?


a. It is the most common lipid abnormality found in peripheral vascular disease
b. It is relatively common in diabetic persons
c. VLDL accumulation characterizes Type IV hyperlipoproteinemia
d. Cholesterol levels are markedly elevated
Answer: a, b, c


Type IV hyperlipoproteinemia is a common condition often found in diabetic persons and is felt to be the most common lipid abnormality found in peripheral vascular disease. VLDL accumulation characterizes Type IV hyperlipidemia which results from an over production of VLDL rather than a clearance defect. VLDL is triglyceride rich lipoproteins produced by the liver and possibly by the intestines.




131. Which of the following is/are true with respect to hypertension?


a. There is a threshold effect of blood pressure on the risk of cardiovascular complications
b. The risk of hypertension is essentially confined to stroke
c. Common antihypertensive regimens may have adverse effects on a patient’s lipid profile
d. Aggressive blood pressure reduction in patients with ischemic heart disease may increase mortality and morbidity
Answer: c


As with the other risk factors there does not appear to be a threshold effect of blood pressure on the risk of cardiovascular complications-even mild elevations convey significant risk. Morbidity of hypertension is strongly correlated with the risk of developing stroke but hypertension also causes peripheral and coronary atherosclerosis. Unfortunately both beta blockers and thiazide diuretics can adversely affect lipid profiles. Overly aggressive lowering of blood pressure in patients with ischemic heart disease has been associated with increased mortality. Nevertheless blood pressure reduction has been shown to be a significant factor in decreasing cardiovascular events in long term follow-up.




132. Which of the following is/are true regarding treatment of diabetes?


a. Strict control with insulin but not oral hypoglycemic agents markedly reduces the incidence of cardiovascular complications in diabetic persons
b. Vascular complications are directly proportional to the degree of glycemic control
c. The effects of diabetes are most marked in individuals with other risk factors
d. The impact of diabetes and cardiovascular risk is relatively uniform
Answer: c


The effect of diabetes is most marked in societies and groups in which the prevalence of atherosclerosis is high even in the absence of diabetes. The impact is not uniform but rather seems to parallel the population in which the diabetic person finds himself augmenting or facilitating the effects of other risk factors. There is little evidence that oral hypoglycemic agents or insulin treatment reduces the incidence of cardiovascular sequela in diabetic persons. Further most diabetics have other appreciable risk factors for atherosclerosis and diabetic persons with optimal levels of other risk factors appear to have little excess risks.




133. Which of the following is/are true regarding cholesterol?


a. Cholesterol ester rich LDLs are removed from the circulation primarily by the liver utilizing the apo B-E receptor
b. Receptor mediated clearance reduces de novo cholesterol synthesis
c. Elevations of LDL alone constitute Type IIa familial hypercholesterolemia
d. Elevated LDLs and VLDLs constitute Type IIb (familial combined)
Answer: a, b, c, d


Receptor mediator clearance of cholesterol rich LDL are accomplished by means of apo B-E receptors. This clearance reduces the de novo production of cholesterol by inhibiting the activity of hydroxymethylglutaryl co-enzyme A (HMG CoA) which catalyzes the rate limiting step in cholesterol synthesis. Isolated LDL elevations constitute Type IIa hypercholesterolemia whereas elevated LDLs and VLDLs constitute Type IIb. Both are associated with coronary artery disease and peripheral vascular lesions. Type IIa exists in heterozygous form in the population in 1:500 individuals and Type IIb may occur in as many as 1:50 people.




134. Which of the following vascular complications are more common in smokers?


a. Coronary artery disease
b. Peripheral vascular occlusive disease
c. Cerebrovascular disease
d. Abdominal aortic aneurysms
Answer: a, b, c, d


Symptoms of intermittent claudication are 15 times more likely to develop in male smokers than nonsmokers and 7 times more likely in women smokers than nonsmokers. Disease progression and graft failure are more common in smokers than nonsmokers and there is an 11 fold increased risk of amputation among smokers. Smokers are at an increased risk for stroke compared to nonsmokers, the risk is approximately double if an individual smokes 40 cigarettes per day. In women smokers the risk is even greater being fully 3.7 times the risk of women who do not smoke and the risk of tobacco and oral contraceptives are particularly dangerous with an almost 22-fold increased risk of stroke. Finally, autopsy series have demonstrated an 8-fold increase in the incidence of aortic aneurysms in those who smoke cigarettes compared to those who did not and smokers have a 2–3-fold increase in the incidence of death from abdominal aortic aneurysm.




135. Which of the following is/are true with respect to exercise training and claudication?


a. The effect is beneficial, real and quantifiable
b. It is secondary to increased blood flow due to collateral development
c. There is an improved metabolic efficiency after exercise training
d. It is associated with significant reduction in blood viscosity and red cell aggregation
Answer: a, c, d


For years exercise training has recognized as having a definite beneficial and quantifiable effect on claudicants. Originally ascribed to increases in blood flow due to development of collateral circulation it is now recognized that this does not occur and rather the improvement relates to increased oxygen extraction and improvement in metabolic efficiency. Recent studies have also documented significant reduction in blood viscosity and red cell aggregation.




136. Which of the following is/are true regarding smoking cessation programs?


a. Since the time of the first Surgeon General’s report the prevalence of smoking among adults decreased from 40% to 29%
b. Using smoking cessation strategies initial success rates are approximately 45%, however, recidivism rates average 40–50%
c. Pharmacologic interventions combined with behavioral counseling increase the likelihood of long term abstinence from smoking
d. Quitting smoking even after prolonged use has beneficial effects on the outcome of peripheral vascular disease
Answer: a, b, c, d


Smoke is a complex substance with more than 2,000 recognized constituents and smoking addiction is a complex behavior. The initial Surgeon General’s report and subsequent efforts have resulted in a substantial decrease in the prevalence of smoking among adults. More than 90% of current smokers state that they would like to stop smoking. Initial success with smoking cessation programs averages 45%, however, the rate of recidivism is high (approximately 50%). Combined pharmacologic and behavioral modification strategies seem to have the best long term results. There is overwhelming evidence that cessation of smoking produces beneficial effects on any outcome measure for peripheral vascular disease. Improvement in claudication distance, avoidance of rest pain, more favorable outcome after arterial reconstructions, a reduced risk of stroke and a decrease in the rate of progression of carotid artery plaque has been demonstrated in those who stop smoking.




137. Which of the following statements is/are true regarding diabetic vascular disease?


a. Diabetes increases the risk of atherosclerosis at almost all anatomic sites
b. Diabetes affects 1–2% of the population. However, of patients undergoing operation for infrageniculate occlusive disease 65–75% are diabetic
c. Diabetic patients are more likely to have an incomplete pedal arch and occlusive involvement of the metatarsal arteries
d. Aortic involvement in the atherosclerotic process is less common
Answer: a, b, c, d


Diabetes clearly influences the pattern of atherosclerotic disease. Although virtually all anatomic sites have an increased incidence of vascular occlusive disease and the most dramatic effects seem to be in the lower extremity. Infrageniculate occlusive disease is markedly more common in diabetes and involvement of multiple tibioperoneal trunks is the rule. Further, incomplete pedal arch and involvement of the metatarsal arteries is at least three times as common in diabetic persons. Aortic involvement in the atherosclerotic process is only half as common in diabetes as in non-diabetes.




138. Which of the following is/are true with respect to risk factors for atherosclerosis?


a. Risk factors are categorized as behavioral or metabolic
b. For metabolic risk factors there are threshold effects
c. Personal behaviors that increase cardiovascular risk do so by modifying metabolic parameters
d. The two primary behaviors that increase risk of atherosclerosis are consuming a diet high in animal fat and smoking cigarettes
Answer: a, c, d


The term risk factor was first introduced in reports from the Framingham study in 1961. Risk factors are generally characterized as behavioral or metabolic. Far and away the two greatest risks are consuming a diet high in animal fat and smoking cigarettes. Other factors, such as a sedentary life style, aggressive high stress (Type A) behavior and alcohol use are much less clear. The metabolic traits are primarily hyperlipidemia, hypertension, diabetes, and homocystinemia. Neither the metabolic or the behavioral factors appear to have a threshold effect, rather there is a progressive increase in the incidence of coronary heart disease with each level of hypertension, plasma cholesterol and number of cigarettes smoked. With the exception of cigarette smoking which directly injures the arterial wall, personal behaviors that increase cardiovascular risk seem to do so by modifying metabolic parameters. Increased physical activity may decrease the risk by improving the resting blood pressure and lipid profiles. Likewise a reduction in Type A behavior may reduce blood pressure. Finally, dietary modification may result in significant changes in total plasma cholesterol, LDL and HDL.

9 comments:

  1. Good sharing, Knee injuries is a common problem with many causes, from acute injuries to medical conditions. Normally people will believe surgery is the only way to relief knee pain, in fact, there are ways surgery free for relief knee pain. Example like using Unloading bracing technology, having ergo mattress etc. Visit at:
    http://kidbuxblog.com/surgery-free-for-relief-knee-pain/

    ReplyDelete
  2. Vascular Care Center is a specialized medical organization for treatment of vascular and endovascular disease and surgeries by most experienced and skilled surgeons in somajiguda, Hyderabad.
    http://www.vascularcarecenter.com

    ReplyDelete
  3. Thank you for sending me the information. It was exactly what we needed.
    Important Facts About Varicose Veins You Should Know

    ReplyDelete
  4. Dr Suresh Bhagia is a highly qualified, experienced, humble, Versatile, Cardiothoracic and Vascular surgeon based in Gujarat. After training in UK, USA and NZ, He came back to serve his motherland.

    ReplyDelete
  5. Dr Suresh Bhagia is a highly qualified, experienced, humble, Versatile, Cardiothoracic and Vascular surgeon based in Gujarat. After training in UK, USA and NZ, He came back to serve his motherland.

    ReplyDelete
  6. Thanks for Great Article, for sharing content and such nice information for me. I hope you will share some more content about. Please keep sharing! Tongue Scraper USA

    ReplyDelete

Related Posts Plugin for WordPress, Blogger...
Related Posts Plugin for WordPress, Blogger...